CHAPTER 7

Reading Comprehension

It can be inferred from the passage that supporters of the Alvarez and Courtillot theories would hold which of the following views in common?

Above is a typical Reading Comprehension question. For now, don’t worry that we haven’t given you the passage this question refers to. In this chapter, we’ll look at how to apply the Kaplan Method to this question, discuss the types of questions the GMAT asks about reading passages, and go over the basic principles and strategies that you want to keep in mind on every Reading Comprehension question. But before you move on, take a minute to think about what you see in the structure of this question and answer some questions about how you think it works:

PREVIEWING READING COMPREHENSION

What Does It Mean to Draw an Inference from a GMAT Passage?

You may remember from the Critical Reasoning chapter the definition of an “inference” on the GMAT: an inference is something that must be true, based on the information provided. There are two important parts to this definition:

(1) A valid GMAT inference must be true. This sets a high standard for what you consider a valid inference. It is sometimes difficult to determine whether a statement in an answer choice must always be true, but you can also approach these questions by eliminating the four answer choices that could be false. Keep both of these tactics in mind for questions that ask for an inference.

(2) A valid GMAT inference is based on the passage. By definition, an inference won’t be explicitly stated in the passage; you will have to understand the passage well enough to read between the lines. But just because it isn’t directly stated doesn’t mean an inference could be anything under the sun. On the GMAT, any inference you draw will be unambiguously supported by something that is stated in the passage. It may take some Critical Thinking to figure out, but you will always be able to pinpoint exactly why a valid inference must be true.

How Much Are You Expected to Know about the Subject Matter of the Passages before You Take the Test?

Familiarity with the subject matter is not required. GMAT passages contain everything you need to answer GMAT questions. In fact, many Reading Comp questions contain wrong answer choices based on information that is actually true but not mentioned in the passage. So if you know the subject, be careful not to let your prior knowledge influence your answer. And if you don’t know the subject, be happy—some wrong answer traps won’t be tempting to you!

Reading Comprehension is designed to test not your prior knowledge but your critical reading skills. Among other things, it tests whether you can do the following:

Note that none of these objectives relies on anything other than your ability to understand and apply ideas found in the passage. This should be a comforting thought: everything you need is right there in front of you. In this chapter, you will learn strategic approaches to help you make the best use of the information the testmakers provide.

What Can You Expect to See in a GMAT Passage That Discusses Two Related Theories?

Because the GMAT cannot ask you purely factual questions that might reward or punish you for your outside knowledge, it tends to focus its questions on the opinions or analyses contained in the passages. Since this question asks you what Alvarez’s and Courtillot’s theories have in common, this means that they must agree on at least one thing. You can anticipate, however, that the two theories are largely not in agreement with each other.

This is a common structure for a GMAT passage: the author discusses more than one explanation of the same phenomenon, describing each in turn or perhaps comparing them directly, usually summarizing the relevant evidence and explaining why disagreement exists among the explanations’ proponents. When a passage contains multiple opinions, keep track of who is making each assertion and how the assertion relates to the other opinions in the passage: does it contradict, agree with, or expand upon what came before? Another important thing to note is whether the author takes a side—does the author prefer one viewpoint to another? Does he offer his own competing argument?

What GMAT Core Competencies Are Most Essential to Success on This Question?

Since the GMAT constructs Reading Comprehension passages in similar ways and asks questions that conform to predictable “types,” you can learn to anticipate how an author will express her ideas and what the GMAT testmakers will ask you about a passage. The more you practice and the more you focus on structure as you read, the stronger your Pattern Recognition skills will become.

Critical Thinking is also essential. As you read, you should ask yourself why the author is including certain details and what the author’s choice of transitional keywords implies about how the ideas in the passage are related.

The best test takers learn how to pay Attention to the Right Detail. Reading Comp passages are typically filled with more details than you could reasonably memorize—and more, in fact, than you will ever need to answer the questions. Since time is limited, you must prioritize the information you assimilate from the passage, focusing on the big picture but allowing yourself to return to the passage to research details as needed.

Practice Paraphrasing constantly as you read, both to keep yourself engaged and to make sure you understand what’s being discussed. Developing this habit will make taking notes much easier, since you’ve already distilled and summarized the most important information in your head. You’ll learn later in this chapter how to take concise and well-organized notes in the form of a Passage Map.

Here are the main topics we’ll cover in this chapter:

QUESTION FORMAT AND STRUCTURE

The directions for Reading Comprehension questions look like this:

Directions: The questions in this group are based on the content of a passage. After reading the passage, choose the best answer to each question. Base your answers only according to what is stated or implied in the text.

In Reading Comp, you are presented with a reading passage (in an area of business, social science, biological science, or physical science) and then asked three or four questions about that text. You are not expected to be familiar with any topic beforehand—all the information you need is contained in the text in front of you. In fact, if you happen to have some previous knowledge about a given topic, it is important that you not let that knowledge affect your answers. Naturally, some passages will be easier to understand than others, though each will present a challenge. The passages will have the tone and content that one might expect from a scholarly journal.

You will see four Reading Comp passages—most likely two shorter passages with 3 questions each and two longer passages with 4 questions each, for a total of approximately 14 questions. However, as is usual for the computer-adaptive GMAT, you will see only one question at a time on the screen, and you will have to answer each question before you can see the next question. The passage will appear on the left side of the screen. It will remain there until you’ve answered all of the questions that relate to it. If the text is longer than the available space, you’ll be given a scroll bar to move through it. Plan to take no longer than 4 minutes to read and make notes on the passage and a little less than 1.5 minutes to answer each question associated with the passage.

TAKEAWAYS: QUESTION FORMAT AND STRUCTURE

 

THE BASIC PRINCIPLES OF READING COMPREHENSION

In daily life, most people read to learn something or to pass the time pleasantly. Neither of these goals has much to do with the GMAT. On Test Day, you have a very specific goal—to get as many right answers as you can. So your reading needs to be tailored to that goal. There are really only two things a Reading Comp question can ask you about: the “big picture” of the passage or its “little details.”

Since the passage is right there on the screen, you don’t need to worry much about the “little details” as you read. (In fact, doing so may hinder your ability to answer questions, as you’ll soon see.) So your main goal as you read is to prepare yourself to get the “big-picture” questions right, while leaving yourself as much time as possible to find the answers to the “little-detail” questions.

Here are the four basic principles you need to follow to accomplish this goal.

Look for the Topic and Scope of the Passage

Think of the topic as the first big idea that comes along. Almost always, it will be right there in the first sentence. It will be something broad, far too big to discuss in the 150–350 words that most GMAT passages contain. Here’s an example of how a passage might begin:

The great migration of European intellectuals to the United States in the second quarter of the 20th century prompted a transmutation in the character of Western social thought.

What’s the topic? The migration of European intellectuals to the United States in the second quarter of the 20th century. It would also be okay to say that the topic is the effects of that migration on Western social thought. Topic is a very broad concept, so you really don’t need to worry about how exactly you word it. You just need to get a good idea of what the passage is talking about so you feel more comfortable reading.

Now, as to scope. Think of scope as a narrowing of the topic. You’re looking for an idea that the author might reasonably focus on for the length of a GMAT passage. If the topic is “the migration of European intellectuals to the United States in the second quarter of the 20th century,” then perhaps the scope will be “some of the effects of that migration upon Western social thought.” It will likely be even more specific: “one aspect of Western social thought affected by the migration.” But perhaps something unexpected will come along. Might the passage compare two different migrations? Or contrast two different effects? Think critically about what’s coming and look for clues in the text that let you know on what specific subject(s) the author intends to focus.

Finding the scope is critically important to doing well on Reading Comp. Many Reading Comp wrong answers are wrong because the information that would be needed to support them is simply not present in the passage. It’s highly unlikely that there will be a “topic sentence” that lays out plainly what the author intends to write about—but the first paragraph probably will give some indication of the focus of the rest of the passage.

Note that some passages are only one paragraph long. In these cases, the topic can still appear in the first sentence. The passage will probably (but not necessarily) narrow in scope somewhere in the first third of the paragraph, as the author doesn’t have much text to work with and needs to get down to business quickly.

Get the Gist of Each Paragraph and Its Structural Role in the Passage

The paragraph is the main structural unit of any passage. At first, you don’t yet know the topic or scope, so you have to read the first paragraph pretty closely. But once you get a sense of where the passage is going, all you need to do is understand what role each new paragraph plays. Ask yourself the following:

Notice that last question—don’t ask yourself, “What does this mean?” but rather, “Why is it here?” Many GMAT passages try to swamp you with tedious, dense, and sometimes confusing details. Consider this paragraph, which might appear as part of a difficult science-based passage:

The Burgess Shale yielded a surprisingly varied array of fossils. Early chordates were very rare, but there were prodigious numbers of complex forms not seen since. Hallucigenia, so named for a structure so bizarre that scientists did not know which was the dorsal and which the ventral side, had fourteen legs. Opabinia had five eyes and a long proboscis. This amazing diversity led Gould to believe that it was highly unlikely that the eventual success of chordates was a predictable outcome.

This is pretty dense stuff. But if you don’t worry about understanding all of the science jargon and instead focus on the gist of the paragraph and why the details are there, things get easier. The first sentence isn’t that bad:

The Burgess Shale yielded a surprisingly varied array of fossils.

A quick paraphrase is that the “Burgess Shale,” whatever that is, had a lot of different kinds of fossils. The passage continues:

Early chordates were very rare, but there were prodigious numbers of complex forms not seen since. Hallucigenia, so named for a structure so bizarre that scientists did not know which was the dorsal and which the ventral side, had fourteen legs. Opabinia had five eyes and a long proboscis.

When you read this part of the passage strategically, asking what its purpose is in context, you see that this is just a list of the different kinds of fossils and some facts about them. There were not a lot of “chordates,” whatever they are, but there was lots of other stuff.

This amazing diversity led Gould to believe that it was highly unlikely that the eventual success of chordates was a predictable outcome.

Notice that the beginning of this sentence tells us why those intimidatingly dense details are there; they are the facts that led Gould to a belief—namely that the rise of “chordates” couldn’t have been predicted. So, on your noteboard, you’d jot down something like this:'

Evidence for Gould’s belief—chordate success not predictable.

Notice that you don’t have to know what any of these scientific terms mean in order to know why the author brings them up. Taking apart every paragraph like this allows you to create a map of the passage’s overall structure. We’ll call this a “Passage Map” from here on—we’ll discuss Passage Mapping in detail later in this chapter. Making a Passage Map will help you acquire a clear understanding of the “big picture.” It will give you a sense of mastery over the passage, even when it deals with a subject you don’t know anything about.

To break down paragraphs and understand the structural function of each part, look for keywords, or structural words or phrases that link ideas to one another. You got an overview of the categories of keywords in the Verbal Section Overview chapter of this book. Let’s now dig a little deeper into how keywords can help you distinguish the important things (such as opinions) from the unimportant (such as supporting examples) and to understand why the author wrote each sentence.

Types of keywords:

As you might have guessed, reading the passage strategically doesn’t mean simply going on a scavenger hunt for keywords. Rather, it means using those keywords to identify the important parts of the passage—its opinions and structure—so you can focus on them and not on little details. Keywords also help you predict the function of the text that follows. Let’s see how this works by taking a look at a simple example. Say you saw a passage with the following structure on Test Day. What kinds of details can you anticipate would fill each of the blanks?

image

You learn about Bob’s attitude toward East Main State Business School through the word excited, which is an emphasis keyword. After because (a logic keyword) will be a reason that East Main State is a great place to go to school or some reason it’s special to Bob. After furthermore and moreover (continuation keywords) will be additional reasons or elaborations of the reason in the first sentence. After however will come some drawback or counterexample that undermines the previous string of good things about East Main State. You may not be able to predict the exact content of the details that fill the blanks, but you can predict the tone and purpose of the details. Reading this way is valuable because the GMAT testmakers are more likely to ask you why the author put the details in, not what’s true about them.

Reading for keywords seems straightforward when the passage deals with subject matter that’s familiar or easy to understand. But what if you were to see the following passage about a less familiar topic? How can you decode the structure of the following paragraph?

image

Here the emphasis keyword important lets you know why the author cares about mirror neurons: they’re important for learning. The details that would fill these blanks are probably dense and intimidating for the non-neurologist, but the strategic reader will still be able to understand the passage well enough to answer the questions correctly. Notice that the structure is identical to the paragraph about Bob, so the details that fill the blanks will serve the same purpose as those you predicted previously. You can anticipate what they will be and why the author is including them. Reading strategically allows you to take control of the passage; you will know where the author is going and what the GMAT will consider important, even if you know nothing about the subject matter of the passage.

Look for Opinions, Theories, and Points of View—Especially the Author’s

An important part of strategic reading is distinguishing between factual assertions and opinions or interpretations. It’s the opinions and interpretations that Reading Comp questions are most often based on, so you should pay the most attention to them. Let’s say you come upon a paragraph that reads:

The coral polyps secrete calcareous exoskeletons, which cement themselves into an underlayer of rock, while the algae deposit still more calcium carbonate, which reacts with sea salt to create an even tougher limestone layer. All of this accounts for the amazing renewability of the coral reefs despite the endless erosion caused by wave activity.

In a sense, this is just like the Burgess Shale paragraph; it begins with a lot of scientific jargon and later tells us why that jargon is there. In this case, it shows us how coral reefs renew themselves. But notice a big difference—the author doesn’t tell us how someone else interprets these facts. He could have written “scientists believe that these polyps account for …,” but he didn’t. This is the author’s own interpretation.

It’s important to differentiate between the author’s own voice and other people’s opinions. GMAT authors may disagree with other people but won’t contradict themselves. So the author of the Burgess Shale passage might well disagree with Gould in the next paragraph. But the author of the Coral Reef passage has laid his cards on the table—he definitely thinks that coral polyps and algae are responsible for the renewability of coral reefs.

Spotting the opinions and theories also helps you to accomplish the goal of reading for structure. Once you spot an idea, you can step back from the barrage of words and use Critical Thinking to dissect the passage, asking, “Why is the author citing this opinion? Where’s the support for this idea? Does the author agree or disagree?”

Consider how you would read the following paragraph strategically:

Abraham Lincoln is traditionally viewed as an advocate of freedom because he issued the Emancipation Proclamation and championed the Thirteenth Amendment, which ended legal slavery in the United States. And indeed this achievement cannot be denied. But he also set uncomfortable precedents for the curtailing of civil liberties.

A strategic reader will zero in on the passage’s keywords and analyze what each one reveals about the structure of the passage and the author’s point of view. Here the keyword traditionally lets you know how people usually think about Lincoln. You might already anticipate that the author is setting up a contrast between the traditional view and her own. Sure enough, the keyword but makes the contrast clear: the author asserts that despite his other accomplishments, Lincoln in fact restricted civil liberties. And the word uncomfortable is an opinion keyword indicating that the author is not at all pleased with Lincoln because of it. However, the author already tempered her criticism with the phrase this achievement cannot be denied, meaning that she won’t go so far as to say that Lincoln was an enemy of freedom.

At this point, the strategic reader can anticipate where the passage’s structure will lead, just as you did on the earlier passages dealing with Bob at East Main State and mirror neurons. Given how this opening paragraph ends, you can predict that the author will spend at least one paragraph describing these “uncomfortable precedents” and how they restricted civil liberties. It might even be possible, since she uses the word precedents, that she goes on to describe how later governments or leaders used Lincoln’s actions as justification for their own restrictions.

This is the power of predictive, strategic reading: by using keywords to anticipate where the author is heading, you will not only stay more engaged as you read, but you’ll also develop a better understanding of the structure of the passage and the author’s point of view—the very things that pay off in a higher GMAT score.

Put together, the passage’s structure and the opinions and theories it contains will lead you to understand the author’s primary purpose in writing the passage. This is critical, as most GMAT passages have a question that directly asks for that purpose. For the Lincoln passage, you might get a question like this:

Which of the following best represents the main idea of the passage?

Just from a strategic reading of the first few sentences, you could eliminate (A) as being a distortion of the first and third sentences, (C) as being too extreme because of the cannot be denied phrase, (D) as irrelevant—either too narrow or just not present in the passage at all, and (E) as missing the author’s big point—that Lincoln helped restrict civil liberties. And just like that, you can choose (B) as the right answer and increase your score.

Don’t Obsess over Details

On the GMAT, you’ll need to read only for short-term—as opposed to long-term—retention. When you finish the questions about a certain passage, that passage is over and done with. You’re promptly free to forget everything about it.

What’s more, there’s certainly no need to memorize—or even fully comprehend—details. You do need to know why they are there so that you can answer big-picture questions, but you can always go back and reread them in greater depth if you’re asked a question that hinges on a detail. And you’ll find that if you have a good sense of the passage’s scope and structure, the ideas and opinions in the passage, and the author’s purpose, then you’ll have little problem navigating through the text as the need arises.

Furthermore, you can even hurt your score by reading the details too closely. Here’s how:

Here’s a great trick for cutting through confusing, detail-laden sentences: focus on the subjects and verbs first, throwing away modifying phrases, and don’t worry about fancy terminology. Let’s revisit some dense text from before:

The coral polyps secrete calcareous exoskeletons, which cement themselves into an underlayer of rock, while the algae deposit still more calcium carbonate, which reacts with sea salt to create an even tougher limestone layer. All of this accounts for the amazing renewability of the coral reefs despite the endless erosion caused by wave activity.

Now look at what happens if you paraphrase these sentences, distilling them to main subjects and verbs, ignoring modifiers, and not worrying about words you don’t understand:

Coral polyps (whatever they are) secrete something … and algae deposit something. This accounts for the amazing renewability of the coral reefs.

The structure of this paragraph has suddenly become a lot more transparent. Now the bulkiness of that first sentence won’t slow you down, so you can understand its role in the big picture.

TAKEAWAYS: THE BASIC PRINCIPLES OF READING COMPREHENSION

 

The basic principles of Reading Comprehension are the following:

Let’s now put all these basic principles together to analyze a passage similar to one you may see on Test Day. However, unlike passages you’ll see on Test Day, the following text has been formatted to approximate the way a strategic reader might see it—important keywords and phrases are in bold, the main ideas are in normal type, and the supporting details are grayed out. Take a moment to read only the bold and regular text: identify what the keywords tell you about the structure, paraphrase the crucial text, and practice predicting what the grayed-out portions contain.

The United States National Park Service (NPS) is in the unenviable position of being charged with two missions that are frequently at odds with one another. Created by an act of Congress in 1916, the NPS is mandated to maintain the country’s national parks in “absolutely unimpaired” condition, while 5somehow ensuring that these lands are available for “the use … and pleasure of the people.” As the system of properties (known as units) managed by the NPS has grown over the years—a system now encompassing seashores, battlefields, and parkways—so has its popularity with the vacationing public. Unfortunately, the maintenance of the system has not kept pace with the record number of visits, and many of the 10properties are in serious disrepair.

Several paths can be taken, perhaps simultaneously, to alleviate the deterioration of the properties within the system. Adopting tougher requirements for admission could reduce the number of additional units that the NPS manages. Congress has on occasion added properties without any input from the NPS itself. 15It is debatable whether all of these properties, which may be of importance to the constituents of individual representatives of Congress, pass the test of “national significance.” Furthermore, some of the units now in the NPS (there are close to 400) receive few visitors, and there is no reason to think that this trend will reverse itself. These units can be removed from the system, and their fates can 20be decided by local public and private concerns. The liberated federal funds could then be rerouted to areas of greater need within the system.

Another approach would be to attack the root causes of the deterioration. Sadly, a great deal of the dilapidated condition of our national parks and park lands can be attributed not to overuse, but to misuse. Visitors should be educated about 25responsible use of a site. Violators of these rules must be held accountable and fined harshly. There are, of course, already guidelines and penalties set in place, but studies strongly indicate that enforcement is lax.

From the first sentence, you can already tell a lot about this passage. The Park Service has an “unenviable” (opinion keyword) dilemma. You can predict that the author will state (in the grayed-out text of the first paragraph) what that dilemma is.

You get more opinion from the author in the final sentence of paragraph 1. “Unfortunately,” (opinion keyword), the Park Service can’t maintain the parks with so many visitors. The problem is apparently “serious” (emphasis keyword). On Test Day you would jot some brief notes on your noteboard about the main idea of the first paragraph:

¶1: NPS problem: lots of visitors = hard to maintain lands

From the first sentence of the second paragraph, where do you anticipate that this passage will go? The phrase “alleviate the deterioration” indicates that the author will discuss some possible solutions to the Park Service’s problem. Now is a good time to ask yourself whether this author is likely to express a preference for one of those solutions in particular. It seems that this author will probably describe the solutions without supporting one over the others, since she says there are “several” and that they might work at the same time. In the middle of the second paragraph, the word “Furthermore” is a continuation keyword, signaling that the paragraph continues to describe a proposed solution and why it might be effective. This is all you need to know about this paragraph from your initial read-through. Take down some notes for later reference:

¶2: Possible solutions; details about the first

Right off the bat, the third paragraph indicates that the author will describe “another approach.” If you need to know details about this other approach, you can always return to the passage to research the answer to a question.

¶3: 2nd solution details

Just from this quick analysis, notice how much you already understand about the structure of the passage and the author’s point of view. You effectively know what the grayed-out parts of the passage accomplish, even though you can’t recite the details they contain. You are now in a strong position to approach the questions that accompany this passage, knowing that you can always return to the passage to clarify your understanding of any relevant details. Let’s look at this first question:

Which of the following best describes the organization of the passage as a whole?

This question asks for the organization of the entire passage. Fortunately, you already have the passage structure in your notes, so there’s no need to go back to the passage itself to answer this question. The author begins by introducing a problem, then advances two potential solutions to that problem. The solutions are complementary, as the author states in line 11 that they can be undertaken “simultaneously.” Choice (D) matches this prediction and is the correct answer.

If you weren’t sure about the answer, you could always eliminate incorrect answer choices by finding the specific faults they contain. (A) cannot be correct because it calls the two solutions “opposing.” (B) is incorrect because the focus of the passage is not on the “factors” leading to the problem but rather on solutions to the problem. (C) is incorrect because the passage gives no “historical survey” of the National Park Service; the only bit of historical data given is the date of its founding (1916). Finally, (E) is incorrect because the passage does not focus on the strengths and drawbacks of a single proposed plan of action; instead, it considers several different suggestions. Moreover, the passage never discusses the flaws of any of these possible solutions. Choice (D) is correct.

Let’s now look at one more question about this passage:

The author is primarily concerned with

This question asks for the author’s purpose in writing the passage. As was the case in the previous question, you’ve already thought through the author’s purpose as you read the passage strategically. Again, this question does not require specific research beyond the notes you’ve already taken.

The author is concerned with advancing two possible solutions to the dilemma faced by the National Park Service. Choice (E) matches your prediction perfectly and is correct.

(A) is incorrect because only one problem is discussed, not “various” problems. (B) is incorrect because it is the solutions, not the causes, of the National Park Service’s problem that are the focus of the passage. (C) is incorrect because the author is not critical of the NPS. Indeed, the word “unenviable” in the first sentence actually signals sympathy with the NPS’s plight. (D) is incorrect because there is no discussion of funding in the passage. (E) is the correct answer.

THE KAPLAN METHOD FOR READING COMPREHENSION

Many test takers read the entire passage closely from beginning to end, taking detailed notes and making sure that they understand everything, and then try to answer the questions from memory. But this is not what the best test takers do.

The best test takers attack the passages and questions critically in the sort of aggressive, energetic, and goal-oriented way you’ve learned earlier. Working this way pays off because it’s the kind of pragmatic and efficient approach that the GMAT rewards—the same type of approach that business schools like their students to take when faced with an intellectual challenge.

The Kaplan Method for Reading Comprehension

1. Read the passage strategically.

2. Analyze the question stem.

3. Research the relevant text.

4. Make a prediction.

5. Evaluate the answer choices.

To help this strategic approach become second nature to you, Kaplan has developed a Method that you can use to attack each and every Reading Comp passage and question.

Step 1: Read the Passage Strategically

Like most sophisticated writing, the prose you will see on the GMAT doesn’t explicitly reveal its secrets. Baldly laying out the why and how of a passage up front isn’t a hallmark of GMAT Reading Comprehension passages. And even more important (as far as the testmakers are concerned), if the ideas were blatantly laid out, the testmakers couldn’t ask probing questions about them. So to set up the questions—to test how you think about the prose you read—the GMAT uses passages in which authors hide or disguise their reasons for writing and challenge you to extract them.

This is why it’s essential to start by reading the passage strategically, staying on the lookout for structural keywords and phrases. With this strategic analysis as a guide, you should construct a Passage Map—a brief summary of each paragraph and its function in the passage’s structure. You should also note the author’s topic, scope, and purpose. Start by identifying the topic and then hunt for scope, trying to get a sense of where the passage is going, what the author is going to focus on, and what role the first paragraph is playing. As you finish reading each paragraph, jot down a short note about its structure and the role it plays in the passage. This process is similar to how you took notes on each paragraph of the National Park Service passage above. When you finish reading the passage, double-check that you got the topic and scope right (sometimes passages can take unexpected turns) and note the author’s overall purpose.

The topic will be the first big, broad idea that comes along. Almost always, it will be right there in the first sentence. There’s no need to obsess over exactly how you word the topic; you just want a general idea of what the author is writing about so the passage gets easier to understand.

The scope is the narrow part of the topic that the author focuses on. If the author expresses his own opinion, then the thing he has an opinion about is the scope. Your statement of the scope should be as narrow as possible while still reflecting the passage as a whole. Your scope statement should also answer the question “What about this topic interests the author?” Identifying the scope is crucial because many wrong answers are unsupported by the facts the author has actually chosen to provide. Remember that even though the first paragraph usually narrows the topic down to the scope, there probably won’t be a “topic sentence” in the traditional sense.

The purpose is what the author is seeking to accomplish by writing the passage. You’ll serve yourself well by picking an active verb for the purpose. Doing so helps not only by setting you up to find the right answer—many answer choices contain active verbs—but also by forcing you to consider the author’s opinion. Here are some verbs that describe the purpose of a “neutral” author: describe, explain, compare. Here are some verbs for an “opinionated” author: advocate, argue, rebut, analyze.

After you finish reading, your Passage Map should look something like this example:

image

You don’t want to take any more than four minutes to read and write your Passage Map. After all, you get points for answering questions, not for creating nicely detailed Passage Maps. The more time you can spend working on the questions, the better your score will be. But creating a Passage Map and identifying the topic, scope, and purpose will prepare you to handle those questions efficiently and accurately.

It generally works best to create your Passage Map paragraph by paragraph. Don’t write while you’re reading, since you’ll be tempted to write too much. But it’s also not a good idea to wait until you’ve read the whole passage before writing anything, since it will be more difficult for you to recall what you’ve read. Analyze the structure as you read and take a few moments after you finish each paragraph to summarize the main points. Include details that are provided as evidence only when keywords indicate their importance. A line or two of paraphrase is generally enough to summarize a paragraph.

Your Passage Map can be as elaborate or as brief as you need it to be. Don’t waste time trying to write out entire sentences if fragments and abbreviations will do. And notice in the above example how effective arrows can be. For example, there’s no sense writing out the purpose as “Describe two solutions to the National Park Service’s dilemma resulting from the dual imperative to keep lands unimpaired and also to allow for their recreational use,” when some simple notes with arrows are just as helpful to you.

Step 2: Analyze the Question Stem

Only once you have read the passage strategically and jotted down your Passage Map should you read the first question stem. The second step of the Kaplan Method is to identify the question type; the most common question types are Global, Detail, Inference, and Logic. We will cover each of these question types in detail later in this chapter. For now, know that you will use this step to ask yourself, “What should I do on this question? What is being asked?” Here are some guidelines for identifying each of the main question types:

In addition to identifying the question type, be sure to focus on exactly what the question is asking. Let’s say you see this question:

The passage states which of the following about the uses of fixed nitrogen?

Don’t look for what the passage says about “nitrogen” in general. Don’t even look for “fixed nitrogen” alone. Look for the uses of fixed nitrogen. (And be aware that the GMAT may ask you to recognize that application is a synonym of use.)

Finally, the GMAT occasionally asks questions that do not fall into one of the four major categories. These outliers make up only about 8 percent of GMAT Reading Comp questions, so you will probably see only one such question or maybe none at all. If you do see one, don’t worry. These rare question types usually involve paraphrasing or analyzing specific points of reasoning in the passage. Often they are extremely similar to question types you know from GMAT Critical Reasoning. Because you can use your Passage Map to understand the passage’s structure and Kaplan’s strategies for Critical Reasoning to deconstruct the author’s reasoning, you will be prepared to handle even these rare question types.

Step 3: Research the Relevant Text

Since there just isn’t enough time to memorize the whole passage, you shouldn’t rely on your memory to answer questions. Treat the GMAT like an open-book test, knowing you can return to the passage as needed. However, don’t let that fact make you overreliant on research in the passage. Doing so could lead to lots of rereading and wasted time. For some question types, you are just as likely to find all the information you need to answer the question correctly using only your Passage Map. Here is how you should focus your research for each question type:

Step 4: Make a Prediction

As you have seen in the Critical Reasoning chapter of this book, predicting the answer before you look at the answer choices is a powerful strategy to increase your efficiency and accuracy. The same is true for GMAT Reading Comp. Making a prediction allows you to know what you’re looking for before you consider the answer choices. Doing so will help the right answer jump off the screen at you. It will also help you avoid wrong answer choices that might otherwise be tempting. Here is how you should form your prediction for each question type:

Step 5: Evaluate the Answer Choices

Hunt for the answer choice that matches your prediction. If only one choice matches, it’s the right answer.

If you can’t find a match for your prediction, if more than one choice seems to fit your prediction, or if you weren’t able to form a prediction at all (this happens for some open-ended Inference questions), then you’ll need to evaluate each answer choice, looking for errors. If you can prove four answers wrong, then you can confidently select the one that remains, even if you aren’t completely sure what you think about it. This is the beauty of a multiple-choice test—knowing how to eliminate the four wrong answers is as good as knowing how to identify the correct one.

Here are some common wrong answer traps to look out for:

Look out for unsupported answer choices and for “half-right/half-wrong” choices, which are fine at the beginning but then take a wrong turn. Some answer choices are very tempting because they have the correct details and the right scope, but they have a not, doesn’t, or other twist that flips their meanings to the opposite of what the question asks for. Watch out for these “180s.”

Now let’s apply the Kaplan Method to an actual GMAT-length passage and some of its questions. One of the questions that follow is the same one you saw at the beginning of the chapter. Read the passage strategically and practice making a Passage Map. You can try your hand at the questions and then compare your analysis to ours, or you can let our analysis below guide you through the steps of the Kaplan Method. For now, don’t worry if you’re not quite sure how to identify the question types; we will cover those thoroughly later in this chapter. For now, concentrate on analyzing what the question asks of you and using the Method to take the most efficient path from question to correct answer.

Questions 1–2 are based on the following passage

Since 1980, the notion that mass extinctions at the end of the Cretaceous period 65 million years ago resulted from a sudden event has slowly gathered support, although even today there is no scientific consensus. In the Alvarez scenario, an asteroid struck the earth, creating a gigantic crater. Beyond the immediate effects 5of fire, flood, and storm, dust darkened the atmosphere, cutting off plant life. Many animal species disappeared as the food chain was snapped at its base.

Alvarez’s main evidence is an abundance of iridium in the KT boundary, a thin stratum dividing Cretaceous rocks from rocks of the Tertiary period. Iridium normally accompanies the slow fall of interplanetary debris, but in KT boundary 10strata, iridium is 10–100 times more abundant, suggesting a rapid, massive deposition. Coincident with the boundary, whole species of small organisms vanish from the fossil record. Boundary samples also yield osmium isotopes, basaltic sphericles, and deformed quartz grains, all of which could have resulted from high-velocity impact.

15Paleontologists initially dismissed the theory, arguing that existing dinosaur records showed a decline lasting millions of years. But recent studies in North America, aimed at a comprehensive collection of fossil remnants rather than rare or well-preserved specimens, indicate large dinosaur populations existing immediately prior to the KT boundary. Since these discoveries, doubts about 20theories of mass extinction have lessened significantly.

Given the lack of a known impact crater of the necessary age and size to fit the Alvarez scenario, some scientists have proposed alternatives. Courtillot, citing huge volcanic flows in India coincident with the KT boundary, speculates that eruptions lasting many thousands of years produced enough atmospheric debris to cause 25global devastation. His analyses also conclude that iridium in the KT boundary was deposited over a period of 10,000–100,000 years. Alvarez and Asaro reply that the shock of an asteroidal impact could conceivably have triggered extensive volcanic activity. Meanwhile, exploration at a large geologic formation in Yucatan, found in 1978 but unstudied until 1990, has shown a composition consistent with 25extraterrestrial impact. But evidence that the formation is indeed the hypothesized impact site remains inconclusive.

  1. It can be inferred from the passage that supporters of the Alvarez and Courtillot theories would hold which of the following views in common?

    • The KT boundary was formed over many thousands of years.

    • Large animals such as the dinosaurs died out gradually over millions of years.

    • Mass extinction occurred as an indirect result of debris saturating the atmosphere.

    • It is unlikely that the specific cause of the Cretaceous extinctions will ever be determined.

    • Volcanic activity may have been triggered by shock waves from the impact of an asteroid.

  2. The author mentions “recent studies in North America” (lines 16–17) primarily in order to

    • point out the benefits of using field research to validate scientific theories.

    • suggest that the asteroid impact theory is not consistent with fossil evidence.

    • describe alternative methods of collecting and interpreting fossils.

    • summarize the evidence that led to wider acceptance of catastrophic scenarios of mass extinction.

    • show that dinosaurs survived until the end of the Cretaceous period.

Step 1: Read the Passage Strategically

Here’s an example of how the passage should be analyzed. We’ve reprinted the passage as seen through the lens of strategic reading. On the left is the passage as you might read it, with keywords and important points in bold. On the right is what you might be thinking as you read.

  PASSAGE

Analysis

the notion that mass extinctions at the end of the Cretaceous period 65 million years ago resulted from a sudden event has slowly gathered support, although even today there is no scientific consensus. In the Alvarez scenario, [bunch of details about an asteroid]

The first sentence is rich with information for the strategic reader. Not only do you get the topic (mass extinctions at the end of the Cretaceous period—whatever that is), but you also get an idea (the notion that [the mass extinction] resulted from a sudden event), the fact that some people agree with it (slowly gathered support), and the fact that not everyone does (no scientific consensus). You can predict that the passage will go on to talk not only about the support but also about why not everyone agrees. You also get one specific theory (the Alvarez scenario) and an elaborate description of what that is—it seems to involve an asteroid.

Alvarez’s main evidence is [lots of detail]

The keywords are pretty clear—here’s some evidence in support of one “sudden event” theory.

You don’t need to focus too much on what the evidence is until there’s a question about it.

Paleontologists initially dismissed the theory, arguing that [something] last[ed] millions of years. But recent studies … doubts about theories of mass extinction have lessened significantly.

With dismissed the theory, it’s clear that this paragraph shows why some would oppose the “sudden event” idea (just as you predicted). Note that millions of years, not normally a keyword, creates contrast with “sudden event.” Then the keyword But announces a change in direction: the recent studies lend support for the “sudden event” theory, so doubts have lessened significantly.

Given the lack of [evidence] to fit the Alvarez scenario, some scientists have proposed alternatives. Courtillot, citing [evidence], speculates that eruptions … cause[d] global devastation. His analyses also conclude [something about iridium]. Alvarez and Asaro replyBut evidenceremains inconclusive.

Alvarez lacks some evidence still, so there are some other theories. Courtillot says something about volcanoes and iridium. It looks like Alvarez makes a counterargument to Courtillot, too. But there isn’t enough evidence either way, so the author doesn’t pick a “winner.”

Your Passage Map would look something like this:

image

This isn’t the only way to word the Passage Map, of course. Anything along these lines would work—so long as you note that there are two theories and that the author doesn’t ultimately prefer one to the other.

Step 2: Analyze the Question Stem

Now it’s time to move to the questions and identify the question type. Question 1 is what you will learn to call an Inference question: it uses the phrase can be inferred. Luckily, this Inference question contains clues that point you to a specific part of the passage. This will save you a lot of time. Question 2 is a Logic question. The phrase in order to indicates that you’re asked to identify why the author includes this detail.

Step 3: Research the Relevant Text

Question 1 asks you to find something that must be true according to both the Alvarez and the Courtillot theories. Our Passage Map shows that the Alvarez theory takes all of paragraph 2 and some of paragraph 4. That’s too much to reread closely. But the Courtillot theory is mentioned only once, in paragraph 4 lines 22–26. Probably the most efficient way to research this question is to read through those two sentences and then deal with paragraph 2, doing focused research on each answer choice as needed.

Question 2 refers to lines 16–17, which are in paragraph 3. Since this is a Logic question, it’s best to begin your research with the Passage Map. Here’s what the Map has to say about paragraph 3:

¶3: Initial disagreement, recent studies, now less doubt

So the “recent studies in North America” are either part of the initial disagreement or a reason that theories of mass extinction are less doubted now. Already, the word recent suggests the latter. But if you weren’t confident about that, you could read the context of lines 16–17. The keywords initially dismissed the theory [of mass extinction] from the sentence before and from the sentence after (Since these discoveries, doubts about theories of mass extinctions have lessened significantly.) seal the deal.

Step 4: Make a Prediction

The answer to Question 1, as you know, must be consistent with both theories. Since you researched Courtillot’s theory, you can quickly eliminate any answer choice that disagrees with it.

Question 2 asks why the “recent studies in North America” were mentioned. Your research shows you that they provide the evidence that reduces doubt about theories of mass extinction. So an easy prediction would be something like “reasons why theories of mass extinction are doubted less than they used to be.”

Step 5: Evaluate the Answer Choices

Question 1(A) says that the KT boundary was formed over thousands of years, and that’s consistent with Courtillot (line 26). What about Alvarez? Scanning through paragraph 2 for anything about “KT boundary” and time, you read in lines 9–11: “KT boundary … rapid, massive deposition.” The word rapid is the only time signal at all, and it hardly fits with many thousands of years. Eliminate (A). Perhaps you could eliminate (B) right away if you remembered that both theories are in the “sudden event” camp. But if not, your Passage Map saves the day—dinosaurs died out gradually over millions of years fits with dinosaur records showed a decline lasting millions of years in paragraph 3. But your Map shows that to be evidence against Alvarez, not in support. So (B) is gone.

Debris saturating the atmosphere is consistent with Courtillot (line 24). What about Alvarez? Line 9 says slow fall of interplanetary debris. So both theories involve debris. Does it saturate the atmosphere? Not explicitly. But this is an Inference question, and you shouldn’t expect to see things repeated explicitly from the passage. You do see that there was a massive deposition … and massive would suggest that there’s a lot of this debris, so it’s plausible that it saturated the atmosphere. There’s not enough evidence to rule out (C), so you can leave it alone for now.

(D) is the opposite of what Courtillot (as well as Alvarez) is trying to do, so this is a quick big-picture elimination. (E) is consistent with Alvarez’s reply (lines 26–28). But this is a reply to Courtillot’s theory—not his theory itself. The passage gives no indication of whether Courtillot would agree with this claim, so (E) is eliminated. Only (C) is left standing, so it must be right.

Note that you can prove (C) to be the right answer without proving why it’s right. To prove why (C) is correct, you’d not only have to connect all the dots in paragraph 2, you’d have to tie it all to mass extinctions by going back to details from paragraph 1. It’s much more efficient to eliminate (A), (B), (D), and (E).

Question 2—Your predicted answer from Step 4 (“reasons why theories of mass extinction are doubted less than they used to be”) leads directly to (D), the correct answer.

(A) sounds nice on its own, but the context has nothing to do with the benefits of field research in a specific way. (B) is the opposite of why those studies are introduced—it is in fact the old belief that these studies dispel. (C) and (E) focus on the details in lines 16–17 themselves—(C) in a distorted way—but not on why those details are there.

READING COMPREHENSION QUESTION TYPES

Though you might be inclined to classify Reading Comp according to the kinds of passages that appear—business, social science, biological science, or physical science—it’s more effective to do so by question type. While passages differ in their content, you can read them in essentially the same way, employing the same strategic reading techniques for each.

Now that you’re familiar with the basic principles of Critical Reasoning and the Kaplan Method, let’s look at the most common types of questions. Certain question types appear again and again on the GMAT, so it pays to understand them beforehand.

The four main question types on GMAT Reading Comp are Global, Detail, Inference, and Logic. Let’s walk through each of these question types in turn, focusing on what they ask and how you can approach them most effectively.

image

The Approximate Distribution of GMAT Reading Comprehension Questions

Global Questions

Any question that explicitly asks you to consider the passage as a whole is a Global question. Here are some examples:

The correct answer will be consistent with the passage’s topic, scope, purpose, and structure. If you’ve jotted these down on your noteboard (and you should!), then it will only take a few seconds to select the right answer.

The GMAT will probably word the answer choices rather formally, so by “few seconds,” we mean closer to 45 than to 10. But that’s still significantly under your average time per question, meaning that you can spend more time dealing with the trickier Inference questions.

Most wrong answers will either get the scope wrong (too narrow or too broad) or misrepresent the author’s point of view. Be wary of Global answer choices that are based on details from the first or last paragraph. These are usually traps laid for those who wrongly assume that GMAT passages are traditional essays with “topic sentences” and “concluding sentences.”

Often answer choices to Global questions asking about the author’s purpose will begin with verbs; in this case, the most efficient approach is to scan vertically to eliminate choices with verbs that are inconsistent with the author’s purpose. Common wrong answers misrepresent a neutral author by using verbs that indicate an opinionated stance.

TAKEAWAYS: GLOBAL QUESTIONS

 

Detail Questions

Detail questions ask you to identify what the passage explicitly says. Here are some sample Detail question stems:

It would be impossible to keep track of all of the details in a passage as you read through it the first time. Fortunately, Detail stems will always give you clues about where to look to find the information you need. Many Detail stems include specific words or phrases that you can easily locate in the passage. A Detail stem might even highlight a sentence or phrase in the passage itself. Using these clues, you can selectively reread parts of the passage to quickly zero in on the answer—a more efficient approach than memorization.

So between highlighting, references to specific words or phrases, and your Passage Map, locating the detail that the GMAT is asking about is usually not much of a challenge. What, then, could these questions possibly be testing? They are testing whether you understand that detail in the context of the passage. The best strategic approach, then, is to read not only the sentence that the question stem sends you to but also the sentences before and after it. Consider the following question stem:

According to the passage, which of the following is true of the guinea pigs discussed in the highlighted portion of the passage?

Let’s say that the highlighted portion of the passage goes like this:

… a greater percentage of the guinea pigs that lived in the crowded, indoor, heated area survived than did the guinea pigs in the outdoor cages.

If you don’t read for context, you might think that something like this could be the right answer choice:

But what if you read the full context, starting one sentence before?

Until recently, scientists had no evidence to support the hypothesis that low temperature alone, and not other factors such as people crowding indoors, is responsible for the greater incidence and severity of influenza in the late fall and early winter. Last year, however, researchers uncovered several experiment logs from a research facility whose population of guinea pigs suffered an influenza outbreak in the winter of 1945; these logs documented that a greater percentage of the guinea pigs that lived in the indoor, heated area survived than did the guinea pigs in the outdoor cages.

Now you could identify this as the right answer:

By reading not just the highlighted portion of the passage but the information before it, you realize that it’s not necessarily true that guinea pigs always survive better indoors; only a specific subset did. If you had picked an answer that matched the first prediction, you would be distorting the facts—you need to understand the information in context.

Make sure that you read entire sentences when answering Detail questions. Notice here that the question stem sends you to the end of the last sentence, but it’s the “no evidence” and “however” at the beginning of the sentences that form the support for the right answer.

TAKEAWAYS: DETAIL QUESTIONS

 

Inference Questions

Reading Comp Inference questions, like Critical Reasoning Inference questions, ask you to find something that must be true based on the passage but is not mentioned explicitly in the passage. In other words, you need to read “between the lines.” Here are some sample Inference question stems:

Inference questions come in two types. The first uses key phrases or highlighting to refer to a specific part of the passage. To solve this kind of question, find the relevant detail in the passage and consider it in the context of the material surrounding it. Then make a flexible prediction about what the correct answer will state.

Consider the following Inference question, once again asking about the guinea pigs we discussed earlier:

Which of the following is implied about the guinea pigs mentioned in the highlighted portion of the passage?

Just like last time, you want to review the context around the highlighted portion. Doing so, you’ll find that “until recently,” there was “no evidence” that temperature affected the flu; “however,” last year these guinea pig records appeared. The logical inference is not explicitly stated, but you can easily put the information from the two sentences together. The answer will be something like this:

Other Inference questions make no specific references, instead asking what can be inferred from the passage as a whole or what opinion the author might hold. Valid inferences can be drawn from anything in the passage, from big-picture issues like the author’s opinion to any of the little details. But you will probably be able to eliminate a few answers quickly because they contradict the big picture.

Then you’ll investigate the remaining answers choice by choice, looking to put each answer in one of three categories—(1) proved right, (2) proved wrong, or (3) not proved right but not proved wrong either. It’s distinguishing between the second and third categories that will lead to success on Inference questions. Don’t throw away an answer because you aren’t sure about it or “don’t like it.”

If you can’t find material in the passage that proves an answer choice wrong, don’t eliminate it. Since the correct answer to an Inference question is something that must be true based on the passage, you can often find your way to the correct answer by eliminating the four choices that could be false.

TAKEAWAYS: INFERENCE QUESTIONS

 

Logic Questions

Logic questions ask why the author does something—why he cites a source, why he includes a certain detail, why he puts one paragraph before another, and so forth. Another way of thinking of Logic questions is that they ask not for the purpose of the passage as a whole but for the purpose of a part of the passage. As a result, any answer choice that focuses on the actual content of a detail will be wrong.

Here are some sample Logic question stems:

Most Logic questions can be answered correctly from your Passage Map—your written summary of each paragraph and the passage’s overall topic, scope, and purpose. If the question references a detail, as does the first sample question stem above, then you should read the context in which that detail appears as well—just as you should for any Detail or Inference question that references a specific detail.

TAKEAWAYS: LOGIC QUESTIONS

 

“Other” Questions

As you saw from the pie chart earlier, approximately 8 percent of Reading Comp questions will not fall neatly into the four main categories described above. On such questions, strategic reading of the question stem becomes even more important, since you’ll need to define precisely what the question is asking you to do, and you may not be able to identify any specific “pattern” that the question falls into. But even within these rare “Other” question types, you can prepare yourself to see questions of the following varieties.

“Critical Reasoning” Questions

Occasionally in Reading Comp, you might find a question that seems more like one of the common Critical Reasoning question types, such as an Assumption, Strengthen, Weaken, or Flaw question. These questions refer to arguments, just as they do in Critical Reasoning—although unlike in Critical Reasoning, a particular argument will usually be confined to just one paragraph or portion of the Reading Comp passage, rather than the entire passage focusing on a single argument.

When you see questions like this, you should research the portion of the passage that includes the argument referenced by the question stem. Just as you would in Critical Reasoning, identify the conclusion and relevant evidence; then use the central assumption to form a prediction for the right answer.

Parallelism Questions

These questions ask you to take the ideas in a passage and apply them through analogy to a new situation. For example, a passage might describe a chain of economic events that are related causally, such as reduced customer spending leading to a slowdown of industrial production, which in turn leads to the elimination of industrial jobs. A Parallelism question might then ask you the following:

Which of the following situations is most comparable to the economic scenario described the passage?

In this case, the correct answer will describe a scenario that is logically similar to the one in the passage. Don’t look for a choice that deals with the same subject matter as the passage, as such choices are often traps. Parallelism questions are more concerned with structure than substance, so the correct answer will provide a chain of causally connected events, such as water pollution from a factory causing the death of a certain type of algae, which in turn causes a decline in the fish population that relied on that type of algae as its primary food source.

Application Questions

Application questions ask you to identify an example or application of something described in the passage. For instance, if the passage describes a process for realigning a company’s management structure, an Application question could give you five answer choices, each of which describes the structure of a given company. Only the correct answer choice will accurately reflect the process described in the passage.

Application questions function similarly to Parallelism questions, except that whereas Parallelism questions ask you to identify an analogous logical situation (irrespective of subject matter), Application questions ask you to apply information or ideas directly, within the same subject area (rather than by analogy or metaphor).

Question Type Identification Exercise

Answers follow this exercise

Identifying the question type during Step 2 of the Kaplan Method for Reading Comprehension is a chance for you to put yourself in control of the entire process that follows: how to research, what to predict, and, thus, how to evaluate answer choices. Use the Core Competency of Pattern Recognition to analyze keywords in the question stem and determine what the question is asking you to do. Based on this analysis, choose the correct question type from the list of choices.

  1. The passage implies which of the following about XXXXXXX?

    • Global

    • Detail

    • Inference

    • Logic

    • Other

  2. According to the passage, each of the following is true of XXXXXXX EXCEPT

    • Global

    • Detail

    • Inference

    • Logic

    • Other

  3. The author mentions XXXXXXX in order to

    • Global

    • Detail

    • Inference

    • Logic

    • Other

  4. The author’s primary objective in the passage is to

    • Global

    • Detail

    • Inference

    • Logic

    • Other

  5. The author makes which of the following statements concerning XXXXXXX?

    • Global

    • Detail

    • Inference

    • Logic

    • Other

  6. An appropriate title for the passage would be

    • Global

    • Detail

    • Inference

    • Logic

    • Other

  7. Which of the following statements about XXXXXXX can be inferred from the passage?

    • Global

    • Detail

    • Inference

    • Logic

    • Other

  8. Which of the following, if true, would most weaken the theory proposed by XXXXXXX?

    • Global

    • Detail

    • Inference

    • Logic

    • Other

  9. Which of the following best states the central idea of the passage?

    • Global

    • Detail

    • Inference

    • Logic

    • Other

  10. Which of the following situations is most comparable to XXXXXXX as it is presented in the passage?

    • Global

    • Detail

    • Inference

    • Logic

    • Other

  11. The passage provides support for which of the following assertions about XXXXXXX?

    • Global

    • Detail

    • Inference

    • Logic

    • Other

  12. The author indicates explicitly that which of the following has been XXXXXXX?

    • Global

    • Detail

    • Inference

    • Logic

    • Other

  13. The author would most likely agree with which of the following?

    • Global

    • Detail

    • Inference

    • Logic

    • Other

  14. The author refers to XXXXXXX most probably in order to

    • Global

    • Detail

    • Inference

    • Logic

    • Other

  15. Which of the following represents the clearest application of XXXXXXX’s theory as described in the passage?

    • Global

    • Detail

    • Inference

    • Logic

    • Other

  16. The passage is primarily concerned with

    • Global

    • Detail

    • Inference

    • Logic

    • Other

Reading Comprehension Quiz

Answers and explanations at end of chapter

Now it’s time to apply the skills you’ve learned in this chapter to the following Reading Comprehension Quiz. Follow the Kaplan Method consistently so this strategic approach can become second nature by Test Day.

Note that the GMAT will give you three or four questions per passage, drawing those questions adaptively from a larger pool of up to six to eight problems. (The computer considers both your current score level and the distribution of question types you’ve seen so far.) For several passages on this quiz, we’ve given you more than four questions so that you can see the full range of questions the GMAT might ask about a given passage. But remember that on Test Day, you’ll see only three or four.

Furthermore, remember that on Test Day, you can’t take notes on the computer screen or skip forward and back between questions. So challenge yourself now not to take notes in this book (use separate scratch paper or a wet-erase noteboard like the one you’ll use at the testing center) and to do the questions in order.

Questions 1–3 are based on the following passage

Due to the laws and mechanics of the American presidential election system, the plurality winner of a state’s popular vote generally gains that state’s entire electoral vote. Thus, the winner of the national election is not necessarily the most popular candidate. In the 30 presidential elections from 1880 to 2000, there were 5two occasions in which the winner of the presidential election did not win a plurality of the popular votes and ten occasions in which the president chosen by this system did not receive the majority of votes cast. Some critics believe that the electoral process should be replaced by a system that might better choose a president who is the most popular candidate among voters. Proposed alternatives include multiple 10rounds of elections, “approval voting,” and “rank voting.”

In the approval voting system, each voter can cast votes for as many candidates as he or she wishes. A voter can select one candidate whom he or she favors, or a voter who disapproves of certain candidates can vote for every candidate except the ones whom he or she opposes. The Secretary General of the United Nations is elected using approval 15voting, and, in a 1990 statewide referendum using the approval voting system, Oregon voters chose from five options for school financing. The approval voting system, however, can be confusing, and it can, theoretically, work against candidates who take strong stands on the issues and therefore attract disapproval.

Rank voting is known more formally as “Borda voting” after its first known 20proponent, Jean Charles Borda. In this scheme, each voter can rank candidates from first to last. Depending on the number of candidates, each position would represent a number of points. For instance, if there were five candidates, the candidate ranked first would get five points, the candidate ranked second would get four points, and so on. The candidate with the most points in the end would be the winner. Rank voting 25is used in the United States by the Associated Press to choose the best college sports teams, and a variant is used in most Australian elections. The rank system is advocated by a number of noted scholars, but it is criticized by some because the candidate ranked first by a majority of voters can still lose. For example, a candidate who is ranked second by 80 percent of the voters could end up with 30more points than one who is ranked first by 52 percent of the voters.

Although alternative voting systems offer noteworthy alternatives to the current process, there is not enough support for an alternative system in the United States to make its adoption likely in the near future on a national scale.

  1. Which of the following best describes the main function of the first paragraph?

    • To lament the nondemocratic nature of the U.S. presidential election system, and to provide alternatives to elections as a means of selecting presidents

    • To show that the winner of a U.S. presidential election isn’t necessarily the most popular candidate, and to explain that some critics suggest modifying the system

    • To describe alternatives for the current U.S. presidential election system, and to advocate the acceptance of one alternative

    • To attack the results of U.S. presidential elections, and to warn of the dangers of not adopting an alternative election system

    • To advocate a sweeping reform of the U.S. presidential election system, and to provide evidentiary support for such a reform

  2. It can be inferred from the passage that in a U.S. presidential election, the selection of the candidate most popular among the voters

    • is more important than the other results that might be generated from the use of a particular election system.

    • would be attained through the use of the approval system for presidential elections.

    • has occurred in only a minority of U.S. presidential elections since 1880.

    • is critical to the maintenance of democratic institutions in the United States.

    • might be achieved through multiple rounds of elections.

  3. The primary purpose of the passage as a whole is to

    • advocate a new system for electing presidents in the United States.

    • explain how the winner of the popular vote can lose the electoral vote.

    • argue that the current electoral system should be replaced with a rank voting system.

    • describe alternatives to the current U.S. presidential election system.

    • demonstrate that the current system of electing U.S. presidents is out-of-date.

Questions 4–6 are based on the following passage

The informal sector of the economy involves activities that, in both developed and underdeveloped countries, are outside the arena of the normal, regulated economy and thus escape official recordkeeping. These activities, which include such practices as off-the-books hiring and cash payments, occur mainly in service 5industries like construction, hotels, and restaurants. Many economists think that the informal sector is an insignificant supplement to the larger formal economy. They base this belief on three assumptions that have been derived from theories of industrial development. But empirical evidence suggests that these assumptions are not valid.

10All three assumptions are, essentially, hypotheses about the character of the labor process at different levels of development. The first is that informal activities are transitory, being a consequence of the imperfect penetration of modern capitalism into less developed regions, and are thus destined to disappear with the advance of industrialization. The second is that the principal reason for the 15continuing existence of an informal sector is to keep a redundant segment of the labor force alive through jobs invented to fit in the interstices of the modern economy. The third is that the informal sector is primarily a feature of peripheral economies such as those of Latin America, Africa, and most of Asia.

Data collected from both developed and underdeveloped countries, however, 20reveal that the informal sector is neither disappearing with industrialization, limited in function to being an employment alternative of last resort, nor exclusively or particularly a Third World phenomenon. Informal sector employment rates in many countries have remained constant in the postwar era. Moreover, if the informal economy were exclusively a refuge from destitution, two facts would logically follow: 25first, average income levels among the informally employed would be significantly lower than among workers in formal activities; second, those who found employment in formal activities would never leave voluntarily in order to move into the informal economy. But neither of these predictions is supported by data.

  1. The primary purpose of this passage is to

    • compare the economies of developed and underdeveloped countries.

    • suggest that economists should develop a new framework for studying the global economy.

    • criticize the notion that the informal economy is primarily a Third World phenomenon.

    • dispute a widely held view of the relationship between the formal and informal sectors of the world economy.

    • argue that workers in the informal economy are almost as well paid as those in the formal economy.

  2. From the passage, which of the following can be inferred about employment opportunities in the informal economy?

    • Workers employed in the informal economy generally receive much lower wages than do workers in the formal economy.

    • Some workers have purposely chosen to work in the informal economy rather than in the formal economy.

    • The number of workers employed in the informal economy has remained stable over the last few centuries.

    • Employment opportunities in the informal economy will decline as agriculture is displaced by industry.

    • The informal economy has created no employment for workers who would otherwise be unemployable.

  3. The author states which of the following about activities in the informal economy?

    • They are an insignificant aspect of the global economy.

    • They provide income for workers who are unemployable in the formal economy.

    • They are a consequence of capitalist penetration of less developed economies.

    • They thrive mainly in underdeveloped countries in Asia, Africa, and Latin America.

    • They consist of such unregulated practices as off-the-books hiring and cash payments.

Questions 7–12 are based on the following passage

The Big Bang model posits that the universe was created in a primordial fireball between 10 and 20 billion years ago. The model attempts to reconstruct the behavior of the early universe using only the rules of general relativity, elementary particle physics, and basic thermodynamics. Matching these claims with 5observations of the universe today shows its validity.

The Big Bang produced a universe made almost entirely of hydrogen and helium. For the first few seconds of the universe’s existence, its temperature was so high—over 10 billion degrees—and energetic electrons, neutrinos, and positrons so abundant that subatomic particles such as neutrons and protons were constantly 10transmuted into one another. As the universe cooled, these transmutations ceased. Most of the matter took the form of the most stable particle, the proton, leaving a universe where the ratio of protons to neutrons was about seven to one.

After a few minutes, the temperature of the universe fell to about 1 billion degrees, cool enough for these protons and neutrons to bond and form nuclei. Deuterons were formed as each neutron found a proton partner. Most 15deuterons then joined another deuteron to form a helium nucleus. As the universe continued to expand and cool, its density became too low to allow further fusion into heavier elements, and so almost all the neutrons were incorporated into helium. Only 0.01 percent of the deuterons formed remained unpaired, and an even smaller 20percentage had fused into elements heavier than helium, such as lithium.

Because only one neutron was formed for every seven protons, six-sevenths of all protons remained isolated. These remaining protons formed hydrogen nuclei containing one proton each. The Big Bang model predicts that about three-quarters of the total mass of the universe will be hydrogen and the remaining quarter 25helium. This simple prediction matches observations very well.

  1. The author is primarily concerned with

    • explaining how the evolution of the universe is incompatible with the implications of the Big Bang model.

    • chronicling the events that followed the creation of matter by the Big Bang.

    • explaining how helium was formed in the early life of the universe.

    • illustrating specific propositions of a model and showing how they have been supported by observation.

    • showing how the ratios of protons to neutrons created in the Big Bang dictated the final ratio of hydrogen to helium in the universe.

  2. In the fourth paragraph, the author introduces information about the Big Bang model’s prediction for the molecular composition of the universe in order to

    • obviate the claim that six-sevenths of all protons remained isolated.

    • show that the remaining quarter of the total mass of the universe must be helium.

    • provide evidence for the veracity of the Big Bang model.

    • describe the means by which remaining protons formed hydrogen nuclei.

    • predict the results of a Big Bang.

  3. According to the passage, which of the following was true during the first few seconds of the universe’s existence?

    • The great majority of neutrons were incorporated into helium molecules.

    • The temperature was not high enough to allow for the formation of deuterons.

    • Vast numbers of subatomic particles rapidly metamorphosed into other types of particles.

    • None

    • I only

    • III only

    • II and III only

    • I, II, and III

  4. The author uses the first paragraph to introduce

    • a scientific model that the passage will show to be supported by modern evidence.

    • a theory of the origin of the universe that the passage will describe and attempt to refute.

    • a description of the theoretical basis for a scientific model and the modern evidence undermining it.

    • an attempt to use recent evidence to revive a discarded scientific model.

    • a detailed description of an appealing but unsupported theoretical model.

  5. The author would most likely agree with the position that

    • models of the creation of the universe that were made before the Big Bang model was created failed because they did not take into account the behavior of subatomic particles.

    • no credence can be given to a scientific model unless it is possible to produce supporting data through a series of controlled experiments.

    • the Big Bang was part of a cycle of expansion and contraction in the universe.

    • it is beneficial to use the statistical evidence at hand when judging the veracity of scientific models for which the process cannot be repeated in a laboratory.

    • the simplicity of the Big Bang theory accounts for its accuracy.

  6. According to the passage, a helium nucleus

    • could only be created early in the universe’s history.

    • has a low density due to the cooling and expansion of the universe.

    • is more stable than a lithium nucleus.

    • has four times the mass of a hydrogen nucleus.

    • consists of exactly two neutrons and two protons.

Questions 13–17 are based on the following passage

Parfleche is the French name for the Plains peoples’ hoemskot ’eo—an envelope- shaped rawhide container for storing clothes, food, and personal items. The parfleche served not only as a practical and durable storage solution, but also as a decorative object of spiritual significance. Among certain tribes, most notably 5the Cheyenne and Arapaho, parfleches were decorated by the women’s painting society, whose members among the Cheyenne were known as moneneheo, the Selected Ones. Although similar in their economic and social importance to craftsmen’s guilds in medieval and Renaissance Western Europe, the painting society also had a spiritual or religious nature. The shamanistic society required application for 10admission and held its members to high artistic and moral standards. The society further displayed its importance by defining aspects of Cheyenne wealth and status.

Painting on rawhide was fraught with challenges. If painting was attempted while the prepared hide was too moist, the applied paint bled, but if the hide was too dry, the skin did not absorb the pigments. This restricted the time frame in which 15painting could best be completed, which meant that designs had to be visualized fully before the work started. Moreover, every aspect of creating a parfleche was a sacred act. Each design element, for instance, was a syntagma—a linguistic or visual unit intended to convey meaning—freighted with symbolic referents. For example, diamond shapes represented the ha ’kot, the grasshopper, an abundant 20grass eater itself symbolic of the bison, the sacred source of food, shelter, tools, and clothing. The tools used were also symbolic: the shape of the “flesher” used to prepare the hide represented lightning bolts—emblematic of the masculine essence of spirit. The flesher removed the flesh from the hide, transforming it into a spiritual container that would hold earthly matter (the people’s material goods).

25Even the position of the parfleche in the lodge held symbolic significance. It was stored beneath the bed of older women, not only because they were careful guardians, but also because they were closer to Grandmother Earth, from whose union with the lightning spirit the animals and plants of the middle world came to provide food and shelter. The symbolism of every aspect of the parfleche, therefore, 30from the interpretable design work on its outside to its storage place within the lodge, reflects the Cheyenne belief in a complementary worldview: the blending of the masculine spirit and the feminine physical matter.

  1. According to the passage, one reason why there was a limited time frame in which to paint a prepared rawhide was that

    • the pigments dried quickly and thus had to be applied with speed to avoid cracking.

    • if the hide was too dry, it absorbed too much paint.

    • if the hide was too moist, it rejected the pigments.

    • the designs had to be fully visualized before painting was started.

    • if the hide was too moist, the paint bled.

  2. The main purpose of the passage is to

    • describe a tool used among Native Americans when working with rawhide.

    • rebut a commonly held view about the symbolism of the parfleche for the Plains peoples.

    • analyze the societies of the Cheyenne and Arapaho people.

    • propose a new method for analyzing the use of symbolism in Native American art.

    • discuss the spiritual and symbolic importance of a rawhide container and its decorations to several Plains tribes.

  3. According to the author, the Cheyenne women’s painting society was unlike Western European guilds of the Middle Ages and the Renaissance in that

    • application for membership was required.

    • the group had significant economic standing in the community.

    • the group had significant social standing in the community.

    • the women’s painting society was religious in nature.

    • the society had an influence on social standing and material valuation.

  4. You can most reasonably conclude that the Cheyenne definition of the term moneneheo (line 6) reflects

    • the high status some women enjoyed as artists in Cheyenne culture.

    • the shamanistic spiritual origins of the women’s painting society.

    • the notion that artists were chosen by the gods to perform their tasks.

    • the self-restrictive nature of the women’s painting society.

    • a woman’s skill in using rawhide tools such as the “flesher.”

  5. The author describes the symbolic meanings of the diamond shape most likely in order to

    • indicate how precious the completed parfleche was to its owner.

    • prove that the grasshopper was superior to the bison in Cheyenne religion.

    • illustrate the visual complexity of the abstract forms used in creating a parfleche.

    • provide an example of the many layers of symbolism involved in creating a parfleche.

    • demonstrate the relationship between the symbolic shapes of the tools and the abstract designs used in creating a parfleche.

Questions 18–23 are based on the following passage

Prior to the nineteenth century, both human and animal populations were limited by the finite resources (such as food) to which they had access. When the enormous increases in prosperity ushered in by the Industrial Revolution essentially freed many Western nations from these constraints, scientists of the time expected 5a Malthusian explosion in population. However, an inverse relationship between prosperity and reproduction was soon noted; the average size of families fell. The trend continues to this day and has spread to recently industrialized portions of the world.

Early biologists tried to explain the transition to smaller families by drawing 10comparisons to the animal world. Animals that have many young tend to live in hostile, unpredictable environments. Since the odds against any given offspring’s survival are high, having many offspring increases the chance that at least one or two of them will survive. In contrast, animals that have fewer children but invest more resources in childrearing tend to live in stable, less hostile environments. 15While the young of these “high-investment” species enjoy the benefits of a relatively safe environment, they need to compete with animals whose young are equally unlikely to perish early in life. Therefore, the biologists observed, progeny that have acquired the skills they need to compete while sheltered by a family have an advantage over their less prepared competitors. By analogy, if people living in 20a prosperous environment produced only a few, pampered children, those children would out-compete the progeny of parents who had stretched their resources too widely.

Critics of this theory argue that there are limitations in conflating animal and human behavior. They argue instead that changes in social attitudes are adequate 25to explain this phenomenon. To a family in a society that is tied to the land, a large number of children is a great boon. They increase family income by being put to work early, and usually some can be persuaded to care for their parents into old age. As a society becomes richer, and as physical labor becomes less important, education may extend into the early twenties, making children economically 30unattractive as they now consume family assets rather than produce them. Meanwhile, plans such as pensions and Social Security mitigate the need for children to care for their parents into their dotage.

  1. The primary purpose of the passage is to

    • criticize explanations of human behavior that are based solely on observations taken from the animal world.

    • show why the expected population explosion following the Industrial Revolution did not occur before the Industrial Revolution.

    • demonstrate how family size was influenced by both environmental restraints and social attitudes before and after the Industrial Revolution.

    • present two alternative theories that explain why family size tends to shrink with increased prosperity.

    • argue that studies based on social attitudes are more effective than models based on evolutionary advantages in accounting for demographic patterns.

  2. According to the passage, which of the following is true of a Malthusian explosion in population?

    • Its occurrence has been limited to those areas of the globe that have remained preindustrial.

    • It is inevitable in societies making the transition from an economy based on agriculture to one based on industry.

    • It was predicted by at least some who lived through the Industrial Revolution in the West.

    • Social scientists have only recently reached consensus on the question of why it fails to occur in recently industrialized countries.

    • It was avoided in Western society because the wealth created by industrialization allowed families to support children through extended periods of education.

  3. The last paragraph performs which of the following functions in the passage?

    • It presents an alternate explanation for the phenomenon described in the first paragraph.

    • It criticizes the explanation presented in the second paragraph.

    • It describes how social attitudes change as societies become richer.

    • It explains a phenomenon presented in the second paragraph.

    • It argues that changing social attitudes are sufficient to explain the phenomenon described in the first paragraph.

  4. The passage mentions each of the following as a possible reason average family size might fall in recently industrialized nations EXCEPT:

    • extended periods of education that make children a drain on family resources

    • well-fed, advantaged children who out-compete those in less advantaged families

    • improved social care of the elderly

    • changed social attitudes

    • increased demand for physical laborers in recently industrialized economies

  5. The information in the passage suggests that which of the following animals would be most likely to have many young?

    • A plant eater that lives in drought-susceptible grassland and is fiercely protective of its offspring.

    • An omnivore whose population is restricted to several small islands that are threatened by human encroachment.

    • A meat eater that has no natural predators but must migrate long distances to maintain its supply of food.

    • A scavenger that competes with few other species for territory and food.

    • A filter feeder that is prey for many creatures in the seasonal streams and lakes where it lives.

  6. The author mentions a decrease in the importance of physical labor (line 28) in order to

    • give an example of the factors that may lead to changes in social attitudes toward family size.

    • demonstrate why those who anticipated a Malthusian explosion in population in industrialized Europe were incorrect.

    • show how family structures adjust to meet the demands of a changing economy.

    • rebut the claims of those who argue that there are limitations in conflating human and animal behavior.

    • illustrate how larger families can increase family income.

Questions 24–29 are based on the following passage

Generally, interspecific matings represent an evolutionary dead end, producing sterile offspring, if any at all. For some species of birds, however, such pairings may indeed bring evolutionary advantages to the participants. In the case of the female collared flycatchers of Gotland, three distinct factors may work to make interspecific 5pairings with pied flycatcher males reproductively beneficial.

In many instances, female collared flycatchers nest with male pied flycatchers while continuing to mate with other collared flycatchers, in effect parasitizing the pied flycatchers, who invest in rearing and fledging any offspring. Often, more than half of the offspring raised by interspecific flycatcher pairs are, in fact, not hybrids. 10Furthermore, an estimated 65 percent of the hybrid offspring of the resident pied flycatcher male are male. Because hybrid females are sterile and males are not, this male bias minimizes the primary disadvantage of interspecific matings: sterile offspring. Habitat specialization may be a third mechanism: these pairings tend to occur in the late spring when the coniferous woods favored by the pied flycatcher 15provide a greater availability of food than the deciduous woods where the collared flycatchers tend to live. Together, these factors form a mechanism to improve substantially the reproductive success of female collared flycatchers beyond what would be expected of interspecific mating with pied flycatcher males.

Although all three of these mechanisms appear to act in concert to form a single 20elaborate mechanism specifically evolved to circumvent the usual disadvantages of interspecific mating, studies have shown similar motivations for the behavior of female collared flycatchers mating within the species. According to Professor Siever Rohwer, collared flycatcher females will choose to nest with subordinate collared flycatcher males that inhabit good territory because collared flycatcher females 25must pair-bond in order to be successful in raising offspring. To engender the best offspring, however, the females will continue to copulate with higher-quality collared males with whom they are not paired. Thus, females seem to be nesting with males of any species with the best territories available at the time, but they will continue to mate with more attractive males outside of their pair bonds.

30A highly unusual behavior, interspecific mating seems to provide certain reproductive advantages to the collared flycatcher female. However, it remains unclear whether the mating behavior of female collared flycatchers evolved to circumvent the usual problems with interspecific mating or whether the behavior is simply an extension of how female collared flycatchers behave when mating within 35their own species.

  1. The author’s primary purpose is to

    • criticize the basis of a scientific theory.

    • defend a hypothesis concerning bird-mating behaviors.

    • point out the need for further study of female collared flycatchers.

    • describe two possible explanations for the interspecific mating behavior of female collared flycatchers.

    • defend an unpopular view of a natural phenomenon.

  2. According to the passage, female collared flycatchers’ mating with male pied flycatchers could be explained by any of the following reasons EXCEPT:

    • Food is more available in pied flycatcher territories during the mating season.

    • Male pied flycatchers can help raise offspring successfully, even if the offspring are not theirs.

    • Male pied flycatchers sire more female offspring than do collared flycatcher males, increasing the reproductive success of the female collared flycatcher.

    • Females are known to nest with subordinate males while pursuing extra-pair copulation with higher-quality males.

    • Females enjoy greater reproductive success by pair-bonding with an inferior male than by not pair-bonding at all.

  3. The bias toward male offspring resulting from the mating of collared flycatcher females and pied flycatcher males is presented as evidence that

    • collared flycatcher females that mate with pied flycatcher males have more dominant male offspring.

    • the offspring from extra-pair matings with collared flycatcher males are more frequently male.

    • female flycatchers are not deterred from interspecific pairing by the likelihood of sterile hybrid offspring.

    • males are produced to reduce interspecific inbreeding in future generations.

    • interspecific breeding is normal in all varieties of flycatchers.

  4. It can be inferred from the passage that

    • food resources are an important determinant of success in raising offspring.

    • having 50 percent male offspring is not optimal for collared flycatcher pairs.

    • flycatchers generally mate for life.

    • males do not vary in the benefits they provide to their offspring.

    • over half of all females engage in extra-pair matings.

  5. Professor Rohwer would most likely agree with which of the following statements?

    • All traits related to particular functions have evolved only for those particular functions.

    • Flycatchers represent the best population for studies of bird-mating behavior.

    • Behaviors may appear functional even under conditions other than those under which the behaviors evolved.

    • Evolution has played no role in shaping the behavior of interspecifically paired flycatchers.

    • Hybridization is generally beneficial.

  6. The mating behavior of female collared flycatchers paired with subordinate male flycatchers is offered as

    • an unwarranted assumption behind the adaptive explanation of interspecific matings.

    • an alternative explanation for pair matings of collared females with pied males.

    • evidence supporting the hypothesis of adaption for interspecific breeding.

    • a discredited mainstream explanation for why hybridization is a dead end.

    • proof in support of the theory that collared and pied flycatchers are separate species.

Questions 30–35 are based on the following passage

A regimen of intrauterine AZT (zidovudine) as a means of reducing the chances of HIV transmission from mother to child was first described in a study known as Protocol 076, the results of which were published in the New England Journal of Medicine in November 1994. The trial found that administration of AZT to 5HIV-positive women during pregnancy and delivery, and to their babies after birth, reduced the transmission of HIV to the infants by two-thirds, compared with a placebo. The study was acclaimed as one of the first successful instances of a prophylaxis preventing HIV transmission—particularly “vertical” transmission, or transmission from mothers to infants. The study, prematurely ended so that all the 10subjects on placebos could be switched to AZT, led to the recommendation that all pregnant women with HIV take AZT.

The validity of the study’s results, however, is debatable. Since Protocol 076 examined only women who had fairly high numbers of T-cells—the white blood cells that coordinate immune response—and who had previously taken AZT for less 15than six months, the same regimen might not succeed for other pregnant women with HIV. Moreover, the study administered large doses of AZT without examining whether lower doses, more economically feasible for uninsured or underinsured women, might be effective. Further, the trial design did not account for important variables: in particular, whether the subjects’ viral load—the amount of HIV in their 20bodies—might have contributed to the difference in transmission rates. Since the exact mechanism and timing of vertical transmission of the HIV virus are unknown, it cannot be ruled out that viral load influences rates of vertical transmission.

Also, the study did not consider the long-term impact of AZT, which is toxic, on the babies themselves. There have been few studies of AZT’s effect on HIV-positive 25infants and none on its effects on uninfected babies. Since the majority of the babies born to HIV-positive women are HIV-negative, and since HIV may mutate into drug-resistant strains following a break in medication, the value of a short-term “zap” with AZT becomes suspect.

  1. The author questions the results of Protocol 076 for which of the following reasons?

    • The number of T-cells was not considered in choosing subjects for the study.

    • The trial was discontinued, and the control group of women on placebo was shifted to AZT.

    • Seventy-five percent of babies born to women in the placebo group were free of infection with HIV.

    • Different transmission rates could have been influenced by different amounts of HIV in the women’s bodies.

    • The exact mechanism and timing of HIV transmission are unknown.

  2. The author of this passage would be LEAST likely to challenge the benefits of intrauterine AZT for which of the following HIV-positive women and/or their future children?

    • A woman who has used the regimen successfully in a previous pregnancy

    • A woman whose T-cell count is dangerously low and who plans to continue taking AZT after her baby is born

    • A woman who is matched with the study’s subjects in terms of ethnicity and socioeconomic status

    • A woman who has lost a previous infant to HIV and who wants to be sure that her next child is healthy

    • A woman whose T-cell count is not low and who has not taken AZT before

  3. The passage implies that the “viral loads” of the subjects in the study are significant because

    • all the subjects had relatively high viral loads, so the study’s results may not apply to other groups of women.

    • the viral loads of babies whose mothers had received the placebo were just as high as those of babies whose mothers had received AZT.

    • viral load must be measured if researchers are to understand the exact mechanism and timing of transmission of HIV.

    • the viral loads of women in the placebo group may have been different from those of women in the AZT group.

    • viral load may be a critical factor in determining transmission, but the researchers wrongly assumed that it was the only factor.

  4. The primary purpose of the passage is to

    • document recent problems with HIV and AIDS treatment.

    • expose medical errors caused by careless methodology in an important clinical trial.

    • raise doubts about a course of treatment based on a groundbreaking study.

    • describe the process of vertical transmission of HIV and suggest preventive therapy.

    • evaluate treatment possibilities for pregnant women with HIV.

  5. The author of the passage would be most likely to agree with which of the following statements?

    • Medications harmful to one group of patients are probably not harmful to another.

    • Recommendations based on a conditional experiment must be qualified.

    • The study should not have been prematurely ended.

    • Clinical trials that may harm patients are unethical.

    • No HIV-positive woman should take AZT during pregnancy.

  6. The primary function of the second paragraph is to

    • refute issues.

    • evaluate solutions.

    • describe results.

    • support action.

    • identify problems.

Answer Key

Question Type Identification Exercise: Answers

1.  C

2.  B

3.  D

4.  A

5.  B

6.  A

7.  C

8.  E

9.  A

10.  E

11.  C

12.  B

13.  C

14.  D

15.  E

16.  A

Answer Key

Reading Comprehension Quiz

  1. B

  2. E

  3. D

  4. D

  5. B

  6. E

  7. D

  8. C

  9. C

  10. A

  11. D

  12. E

  13. E

  14. E

  15. D

  16. A

  17. D

  18. D

  19. C

  20. A

  21. E

  22. E

  23. A

  24. D

  25. C

  26. C

  27. A

  28. C

  29. B

  30. D

  31. E

  32. D

  33. C

  34. B

  35. E

Answers and Explanations

Question Type Identification Exercise: Answers

1.  C

When you see “implies” or “suggests,” know that you’re facing an Inference question.

2.  B

When you see direct, categorical language like “according to the passage,” know that you’re facing a Detail question.

3.  D

The phrase “in order to” means that this is a Logic question. Logic questions deal with the author’s motivations. If a question asks you why the author includes a paragraph, brings up a detail, or cites a source, know that you are facing a Logic question.

4.  A

The phrase “primary objective” means that this is a Global question.

5.  B

The phrase “makes which of the following statements” signals that this is a Detail question.

6.  A

The phrase “title for the passage” means that we must base our answer on the whole passage, so this is a Global question.

7.  C

The word “inferred” means that this is an Inference question.

8.  E (Weaken)

The word “weaken” means that this is a Weaken question, one of the rarer question types in Reading Comp.

9.  A

The phrase “central idea of the passage” means that this is a Global question.

10.  E (Parallelism)

The phrase “most comparable to” means that this is a Parallelism question.

11.  C

The phrase “passage provides support for” means that this is an Inference question.

12.  B

The phrase “indicates explicitly” means that this is a Detail question.

13.  C

The phrase “most likely agree” signals an Inference question.

14.  D

The phrase “in order to” means that this is a Logic question.

15.  E (Application)

The phrase “represents the clearest application of” means that this is an Application question. Application questions ask you to find the answer choice that is most consistent with a description, theory, or process described in the passage.

16.  A

The phrase “passage is primarily concerned” means that this is a Global question.

Answers and Explanations

Reading Comprehension Quiz

On the left, we’ve shown how keywords help you to identify the major elements of the passage and its structure and what you could skim over. On the right, we’ve shown what you might be thinking as you read the passage strategically.

Passage for Questions 1–3

Analysis

Due to the laws and mechanics of the American presidential election system, the plurality winner of a state’s popular vote generally gains that state’s entire electoral vote. Thus, the winner of the national election is not necessarily the most popular candidate. In the 30 presidential elections from 1880 to 2000, there were two occasions in which the winner of the presidential election did not win a plurality of the popular votes and ten occasions in which the president chosen by this system did not receive the majority of votes cast. Some critics believe that the electoral process should be replaced by a system that might better choose a president who is the most popular candidate among voters. Proposed alternatives include multiple rounds of elections, “approval voting,” and “rank voting.”

The topic, the American presidential election system, is announced in the first sentence.

A quirky effect of the current system: the winner isn’t necessarily the most popular candidate. This leads the critics to suggest that a different system might be better.

Because the author enumerates the possible alternative systems at the end of the opening paragraph, we expect to see at least some of them discussed in later paragraphs.

In the approval voting system, each voter can cast votes for as many candidates as he or she wishes. A voter can select one candidate whom he or she favors, or a voter who disapproves of certain candidates can vote for every candidate except the ones whom he or she opposes. The Secretary General of the United Nations is elected using approval voting, and, in a 1990 statewide referendum using the approval voting system, Oregon voters chose from five options for school financing. The approval voting system, however, can be confusing, and it can, theoretically, work against candidates who take strong stands on the issues and therefore attract disapproval.

Indeed, here’s the first alternative we’ll hear about: approval voting, which simply seems to be a system allowing voters to cast votes for multiple candidates they approve of.

Here we get a couple of examples of elections run according to approval voting.

The word “however” indicates contrast. We’ve been hearing that approval voting is a system that is in fact in use, but the author believes that it nevertheless has a couple of drawbacks.

Rank voting is known more formally as “Borda voting” after its first known proponent, Jean Charles Borda. In this scheme, each voter can rank candidates from first to last. Depending on the number of candidates, each position would represent a number of points. For instance, if there were five candidates, the candidate ranked first would get five points, the candidate ranked second would get four points, and so on. The candidate with the most points in the end would be the winner. Rank voting is used in the United States by the Associated Press to choose the best college sports teams, and a variant is used in most Australian elections. The rank system is advocated by a number of noted scholars, but it is criticized by some because the candidate ranked first by a majority of voters can still lose. For example, a candidate who is ranked second by 80 percent of the voters could end up with more points than one who is ranked first by 52 percent of the voters.

Now the author proceeds to discuss a second alternative: rank voting. The words “in this scheme” indicate that we’re about to be given a description of the rank voting method.

Here we get specific examples of elections using rank voting.

Like approval voting, rank voting apparently has its pros and cons.

Although alternative voting systems offer noteworthy alternatives to the current process, there is not enough support for an alternative system in the United States to make its adoption likely in the near future on a national scale.

The keyword “Although” signals a contrast: the author thinks there are “noteworthy” methods that could replace the current one, but they aren’t likely to be put into use anytime soon.

Passage Map

¶1: American presidential election system = electoral system
Problem: candidate with most popular vote can lose
Critics favor alternatives

¶2: Approval voting: can vote for several candidates
Drawback: can be confusing

¶3: Rank voting: voters rank choices
Drawback: candidate ranked 1st by majority could still lose

¶4: Author likes alternatives, but they probably won’t be adopted soon

Topic: American presidential election system

Scope: Problem with, and alternatives to, the current system

Purpose: Describe problem with current electoral system and two potential alternatives

1.    (B)

Which of the following best describes the main function of the first paragraph?

Step 2: Analyze the Question Stem

Because we are asked for the function of a chunk of the passage, this is a Logic question. Specifically, we are asked to deduce the role played by the first paragraph.

Step 3: Research the Relevant Text

A glance at our Passage Map will suffice in this case. We shouldn’t have to go back to the passage text itself.

Step 4: Make a Prediction

The first paragraph, we noted in our Passage Map, describes the American presidential election system, states that there is a problem with the current system that has caused it to receive some criticism, and lists several proposed alternatives.

Step 5: Evaluate the Answer Choices

(B) matches our prediction and is the correct answer. The fact that the winner of a U.S. presidential election isn’t necessarily the most popular candidate is the “problem” we recorded in our Passage Map. (A) is incorrect because the author takes a very neutral tone. The initial verb, “to lament,” is sufficient to throw this one out. Moreover, the author never describes the U.S. presidential election system as “nondemocratic.” (C) is incorrect because the author does not “advocate” for either of the proposed alternatives. (D) is incorrect because the author does not “attack” election results. (E) is similar to (C); again, the author does not “advocate” making a change to the current system. Choice (B) is correct.

2.    (E)

It can be inferred from the passage that in a U.S. presidential election, the selection of the candidate most popular among the voters

Step 2: Analyze the Question Stem

The word “inferred” signals that this is an Inference questions. The correct answer must be true based only on information in the passage. Note that the question asks us specifically about the selection of the most popular candidate in U.S. presidential elections.

Step 3: Research the Relevant Text

We can use our Passage Map to confirm that the question stem points to the first paragraph, where the author discusses the current practice used in American presidential elections. We read that under the current system, the winner of the popular vote is not necessarily the winner of an election and that critics have proposed alternative systems that would make the winner more likely to be the popular favorite, including multiple rounds of elections, approval voting, and rank voting.

Step 4: Make a Prediction

While it is difficult to make a specific prediction because the sentence in the question stem could be completed in multiple ways, the correct answer will be in accord with the information we have gleaned from the first paragraph about the U.S. presidential election system.

Step 5: Evaluate the Answer Choices

(A) cannot be correct because there is nothing in the first paragraph to indicate that electing the most popular candidate is “more important” than any other result of a particular election system. (B) might be tempting, because we read about the possibility of using approval voting in the last sentence of the first paragraph. However, we can throw out this choice as extreme. The author merely states that approval voting is one of the systems that the critics believe “might” better choose the most popular candidate; there is no indication that the approval voting system would definitely have this effect. (C) cannot be correct according to the numbers given in the first paragraph. In 20 of 30 presidential elections, the winner has received the majority of votes cast, and in 28 of 30, he has received the plurality of votes cast. Either way, that’s the majority of elections since 1880. (D) is unsupported. There is no mention of anything being “critical to the maintenance of democratic institutions.” That leaves (E), which corrects the extreme language of (B). It is true, according to the last few lines of the first paragraph, that selecting the most popular candidate “might” be achieved through multiple rounds of elections. Choice (E) is correct.

3.    (D)

The primary purpose of the passage as a whole is to

Step 2: Analyze the Question Stem

The phrase “primary purpose” indicates that this is a Global question. We need to ascertain the author’s purpose in writing the passage.

Step 3: Research the Relevant Text

The only research we’ll need to do will be to consult our statements of topic, scope, and purpose.

Step 4: Make a Prediction

We said that the author’s purpose is to describe a problem with the current U.S. electoral system, as well as two alternatives that have been proposed.

Step 5: Evaluate the Answer Choices

(D) matches our prediction and is the correct answer. (A) is incorrect because the author isn’t “advocating” for anything. The tone of the passage is fairly neutral. (B) is a classic wrong answer type called “faulty use of detail.” The passage definitely discusses how the winner of the popular vote can lose the electoral vote, but it goes on to discuss the two proposed alternative election systems. Watch out for answer choices that, like this one, provide you with only part of the passage when you’re looking for a choice that encompasses the entire thing. (C) is incorrect because the author does not make any recommendations. (E) is incorrect because the passage makes no mention of the idea that the current electoral system is “out-of-date.” Choice (D) is correct.

On the left, we’ve shown how keywords help you to identify the major elements of the passage and its structure and what you could skim over. On the right, we’ve shown what you might be thinking as you read the passage strategically.

Passage for Questions 4–6

Analysis

The informal sector of the economy involves activities that, in both developed and underdeveloped countries, are outside the arena of the normal, regulated economy and thus escape official recordkeeping. These activities, which include such practices as off-the-books hiring and cash payments, occur mainly in service industries like construction, hotels, and restaurants. Many economists think that the informal sector is an insignificant supplement to the larger formal economy. They base this belief on three assumptions that have been derived from theories of industrial development. But empirical evidence suggests that these assumptions are not valid.

The author begins by describing the informal sector of the economy, which will be the topic of the passage.

Economists think the informal sector is not important, based on three assumptions.

The author of the passage disagrees with the economists’ assumptions, citing evidence to the contrary.

All three assumptions are, essentially, hypotheses about the character of the labor process at different levels of development. The first is that informal activities are transitory, being a consequence of the imperfect penetration of modern capitalism into less developed regions, and are thus destined to disappear with the advance of industrialization. The second is that the principal reason for the continuing existence of an informal sector is to keep a redundant segment of the labor force alive through jobs invented to fit in the interstices of the modern economy. The third is that the informal sector is primarily a feature of peripheral economies such as those of Latin America, Africa, and most of Asia.

The second paragraph outlines the three assumptions made by the economists mentioned above.

The author’s opinion doesn’t figure into this paragraph—it only identifies the three assumptions.

Data collected from both developed and underdeveloped countries, however, reveal that the informal sector is neither disappearing with industrialization, limited in function to being an employment alternative of last resort, nor exclusively or particularly a Third World phenomenon. Informal sector employment rates in many countries have remained constant in the postwar era. Moreover, if the informal economy were exclusively a refuge from destitution, two facts would logically follow: first, average income levels among the informally employed would be significantly lower than among workers in formal activities; second, those who found employment in formal activities would never leave voluntarily in order to move into the informal economy. But neither of these predictions is supported by data.

Here the author discusses how the “data” refute the first of the economists’ assumptions (transitory)…

 and the second (redundant)…

 and the third (peripheral).

 The author further develops the rebuttal.

Passage Map

¶1: Description of informal sector
Economists think it’s unimportant, due to three assumptions

¶2: Economists’ assumptions about the informal sector:
(1) transitory
(2) created for those who cannot find jobs within the formal economy (redundant)
(3) a feature of developing economies (peripheral)

¶3: Economists’ assumptions are all incorrect
Reasons given to refute the first two

Topic: Informal sector of the economy

Scope: Economists’ belief and assumptions about the informal sector

Purpose: To show the economists are incorrect in their assumptions and thus in their belief that the informal sector is insignificant

4.    (D)

The primary purpose of this passage is to

Step 2: Analyze the Question Stem

The phrase “primary purpose” signals that this is a Global question. We need the author’s reason for writing the passage.

Step 3: Research the Relevant Text

For this question, we’ll simply consult our notes on topic, scope, and purpose. We shouldn’t have to go back to the passage itself.

Step 4: Make a Prediction

We noted that the author’s purpose in writing the passage is to demonstrate that the economists are wrong in thinking the informal sector of the economy to be “insignificant” compared to the formal sector.

Step 5: Evaluate the Answer Choices

(D) matches our prediction. The “widely held view” is that of the economists. (A) is unsupported by passage, which focuses on the economists’ assumptions about the informal sector of the economy. (B) is again unsupported. The author makes no recommendation as to what the economists “should” do. (C) is a classic faulty use of detail. The idea that the informal sector is a Third World phenomenon is certainly mentioned and criticized in the passage (specifically, in paragraph 3), but this isn’t the purpose of the entire passage. (E) is another faulty use of detail. The pay rates of informal- and formal-sector workers are mentioned in the third paragraph, but again, this doesn’t constitute the purpose of the entire passage. Choice (D) is correct.

5.    (B)

From the passage, which of the following can be inferred about employment opportunities in the informal economy?

Step 2: Analyze the Question Stem

The word “inferred” signals an Inference question. This question stem provides us with a context clue: “employment opportunities in the informal economy.”

Step 3: Research the Relevant Text

Unless a question specifically mentions a different point of view, it is asking for the author’s perspective. We can safely confine our research to the first and third paragraphs and leave out the second, which describes the economists’ assumptions. In the first paragraph, we learn that informal sector occupations are typically service jobs such as construction, hotel, or restaurant jobs. In the third paragraph, we learn that informal sector jobs are not jobs “of last resort,” because they don’t necessarily pay a great deal less than formal sector jobs and because some workers actually move “voluntarily” from the formal to the informal sector.

Step 4: Make a Prediction

While it is difficult to make a precise prediction for this question, the correct answer will follow from our research. We’ll check the answer choices against it one at a time.

Step 5: Evaluate the Answer Choices

(A) states the opposite of the wage information we found in the third paragraph. (B) paraphrases the last couple of lines of the passage: data support the idea that some workers do voluntarily move from formal sector employment to informal sector employment. This is the correct answer. (C) is too extreme. We learn in the third paragraph that “informal sector employment rates in many countries have remained constant in the postwar era,” but we have no support for the idea that such employment has remained stable for “centuries.” (C) also confuses employment rates (which are percentages) with the number employed. (D) restates one of the economists’ assumptions. It is the economists, not the author, who believe that the informal sector will disappear with increasing industrialization. (E) is extreme. While the author debunks the economists’ assumption that the informal sector is “limited in function to being an employment alternative of last resort,” she never goes so far as to say that not a single job for an otherwise unemployable person has been created in the informal sector. Choice (B) is correct.

6.    (E)

The author states which of the following about activities in the informal economy?

Step 2: Analyze the Question Stem

This Detail question asks us for something the author “states” explicitly. All we need to do is go back to the passage and look up the “activities in the informal economy.”

Step 3: Research the Relevant Text

The “activities” that take place in the informal economy are mentioned at the beginning of the first paragraph. We learn that they are typically “off-the-books” and that they include hiring and cash payments in the construction, hotel, and restaurant industries.

Step 4: Make a Prediction

We’ll look for an answer choice that mentions any of the details we gleaned from our research. Remember that for a Detail question, the correct answer has to come from the passage nearly verbatim.

Step 5: Evaluate the Answer Choices

(E) restates what we read in the first paragraph and is the correct answer. (A) is incorrect because it is the economists, not the author, who believe that informal sector activities are “insignificant.” (B) is incorrect because, again, it states one of the economists’ assumptions, not the author’s opinion. Similarly, (C) and (D) are incorrect because they present the economists’ beliefs, not the author’s. Choice (E) is correct.

On the left, we’ve shown how keywords help you to identify the major elements of the passage and its structure and what you could skim over. On the right, we’ve shown what you might be thinking as you read the passage strategically.

Passage for Questions 7–12

Analysis

The Big Bang model posits that the universe was created in a primordial fireball between 10 and 20 billion years ago. The model attempts to reconstruct the behavior of the early universe using only the rules of general relativity, elementary particle physics, and basic thermodynamics. Matching these claims with observations of the universe today shows its validity.

The topic appears right away: the Big Bang model.

The word “attempts” begs the question of whether the model succeeds.

Aha! Indeed, the Big Bang model succeeded. And current observations demonstrate this.

The Big Bang produced a universe made almost entirely of hydrogen and helium. For the first few seconds of the universe’s existence, its temperature was so high—over 10 billion degrees—and energetic electrons, neutrinos, and positrons so abundant that subatomic particles such as neutrons and protons were constantly transmuted into one another. As the universe cooled, these transmutations ceased. Most of the matter took the form of the most stable particle, the proton, leaving a universe where the ratio of protons to neutrons was about seven to one.

Lots of details about the Big Bang. Since we don’t want to read details too closely, we could try to get away just with saying “details about the Big Bang” in our map, but then paragraph 3 would be the same thing. So, we have to find some way of grouping these details. The passage gives us two sequences that let us do so easily: time (first few seconds) or temperature (so high and cooled).

After a few minutes, the temperature of the universe fell to about 1 billion degrees, cool enough for these protons and neutrons to bond and form nuclei. Deuterons were formed as each neutron found a proton partner. Most deuterons then joined another deuteron to form a helium nucleus. As the universe continued to expand and cool, its density became too low to allow further fusion into heavier elements, and so almost all the neutrons were incorporated into helium. Only 0.01 percent of the deuterons formed remained unpaired, and an even smaller percentage had fused into elements heavier than helium, such as lithium.

Once again, a ton of details about the Big Bang. If we’re using the time sequence, then this is the next few minutes. If we’re using the temperature sequence, then the universe continues to cool. Happily, we don’t need to understand a thing about particle physics to understand the basic structure of the passage!

Because only one neutron was formed for every seven protons, six-sevenths of all protons remained isolated. These remaining protons formed hydrogen nuclei containing one proton each. The Big Bang model predicts that about three-quarters of the total mass of the universe will be hydrogen and the remaining quarter helium. This simple prediction matches observations very well.

Still more details! These focus on the fact that the model’s predictions match observations, which (as we remember from paragraph 1) is why the model can be said to be valid.

Passage Map

¶1: Introduce Big Bang model of the universe’s creation; it is valid

¶2: First seconds: temperature and energy high

¶3: After a few minutes: cooler, more stable

¶4: Observation confirms BB model

Topic: Big Bang Model

Scope: Validity of (B.B.M.)

Purpose: Demonstrate (the validity of the B.B.M.)

7.    (D)

The author is primarily concerned with

Step 2: Analyze the Question Stem

The phrase “primarily concerned” in the question stem lets us know that this is a Global question.

Step 3: Research the Relevant Text

Using our Passage Map—particularly the topic, scope, and purpose—we should be able to predict the correct answer without having to reread any of the passage.

Step 4: Make a Prediction

From our scope and purpose notes, we can predict that the correct answer will say that the author is “demonstrating the Big Bang model’s validity.”

Step 5: Evaluate the Answer Choices

(D) is the only choice to mention that the passage addresses how observation supports the model’s propositions. (D) is the correct answer. (A) is a 180 trap; it claims that the Big Bang model isn’t valid. Change incompatible to compatible, and it would be correct. (B) focuses on the details from paragraphs 2 and 3 but does not address how those details fit within the passage as a whole. (C) claims that the passage is focused mainly on helium; the word helium certainly shows up a bit in paragraphs 3 and 4, but it isn’t the scope of the passage as a whole. (E) distorts details given in the passage. The author of the passage uses the ratios of hydrogen and helium to show us how well the Big Bang model matches observations. Choice (D) is correct.

8.    (C)

In the fourth paragraph, the author introduces information about the Big Bang model’s prediction for the molecular composition of the universe in order to

Step 2: Analyze the Question Stem

This is a Logic question, because we are asked why the author used a specific piece of information. The question stem gives us a perfect reference—“the Big Bang model’s prediction for the molecular composition of the universe”—to guide our research.

Step 3: Research the Relevant Text

The reference from the question stem leads us to paragraph 4. Our Passage Map paraphrased paragraph 4 as showing that observation confirms the Big Bang model. Note that by doing our research in the Passage Map first, we save ourselves from spending time rereading unnecessarily.

Step 4: Make a Prediction

From our research we can predict that the correct answer will say that the author included the details of the paragraph in order to “show us why the Big Bang model is valid.”

Step 5: Evaluate the Answer Choices

Our prediction matches very nicely with (C); this is the correct answer. The word obviate in (A) means “to avoid or eliminate.” The author believes it’s true that six-sevenths of all protons remained isolated, so (A) doesn’t make sense. (B) mixes things up; the author isn’t using the Big Bang model to prove that the universe is made up of certain kinds of molecules; instead, the author is using the molecular makeup of the universe to argue that the Big Bang model is valid. (D) incorrectly states that the author’s objective in this paragraph is to explain the how of hydrogen nucleus formation. The details of nucleus formation are discussed only to help the author make the case that the Big Bang theory is sound. (E) is wrong because the author, in the fourth paragraph, is not out to predict the results of a Big Bang, but rather to validate the Big Bang model’s predictions. Choice (C) is correct.

9.    (C)

According to the passage, which of the following was true during the first few seconds of the universe’s existence?

Step 2: Analyze the Question Stem

The phrase “according to” in the stem signals a Detail question. So the right answer must be something that the author explicitly states. We also see that we’re looking for something that “was true during the first few seconds” of the universe.

Step 3: Research the Relevant Text

The Passage Map focuses our research on paragraph 2, specifically lines 7–10. We should notice that this sentence emphasizes temperatures that are “so high” and particles “so abundant” that constant change was the order of the day (or, at least, its first few seconds).

Step 4: Make a Prediction

The correct answer will agree with the ideas expressed in lines 7–10. Anything else must be incorrect because that’s the only place we learn about the “first few seconds.”

Step 5: Evaluate the Answer Choices

For Roman numeral questions, we should start with the statement that appears most frequently in the answer choices, which in this case is III. Lines 8–10 provide support for Statement III. Each phrase and idea in Statement III matches directly with a phrase or idea from the passage: “vast numbers” matches with “so abundant,” “rapidly metamorphosed” matches with “constantly transmuted,” and “subatomic particles” matches with, well, “subatomic particles.” Since Statement III is correct, eliminate (A) and (B). Next evaluate Statement II, as it appears in two of the remaining three choices. Researching line 7 shows that the temperature was “high” during the first few seconds of the universe’s existence. So, the phrase “not high enough” kills Statement II. That eliminates (D) and (E), leaving only (C). Choice (C) is the correct answer.

Note that both Statements I and II refer to details from paragraph 3, which the time/sequence keywords show to describe not the first few seconds of the universe’s existence but rather the few minutes thereafter.

10.   (A)

The author uses the first paragraph to introduce

Step 2: Analyze the Question Stem

This question stem asks for the purpose not of the whole passage, but of one of its parts. That makes this a Logic question. The stem makes clear reference to the first paragraph.

Step 3: Research the Relevant Text

As the question focuses on the whole of the first paragraph, our research can be focused on our Passage Map. We wrote that paragraph 1 tells us what the Big Bang model is and that it’s valid, supported by observation of the universe.

Step 4: Make a Prediction

A reasonable prediction would be something like this: the author uses the first paragraph to introduce the Big Bang model and claim that it’s valid.

Step 5: Evaluate the Answer Choices

(A) matches our prediction perfectly. Every other answer choice gets the detail right (“theory” or “model”) but the purpose wrong: “refute,” “undermining,” “discarded,” and “unsupported” are all negative terms, while the author’s only purpose is to be positive. Choice (A) is correct.

11.   (D)

The author would most likely agree with the position that

Step 2: Analyze the Question Stem

The word “agree” makes this an Inference question, so the right answer must be true based on the passage but is likely not mentioned explicitly. This stem contains no reference to a particular detail or paragraph.

Step 3: Research the Relevant Text

When the test asks you an Inference question without providing a particular reference to guide your research, prepare yourself to evaluate the answers by refreshing your memory on the topic, scope, and purpose.

Step 4: Make a Prediction

We cannot anticipate the exact wording of the correct answer, but since we are asked what the author would agree with, we know the right answer will be in line with “The Big Bang model corresponds well with the observed universe; the model seems valid.”

Step 5: Evaluate the Answer Choices

(A) can be eliminated for two reasons. First, the passage only mentions the Big Bang model, not any others. So there’s no way we can state with confidence why any of those other models failed. Second, the author only ever uses positive opinion keywords, such as “validity.” So an answer about why something “failed” can’t be correct. (B) is a 180 wrong answer trap. The author gives credence to the Big Bang model, not by “a series of controlled experiments,” but by “matching … claims with observations.” (C) is incorrect because although the passage does mention “expansion” in line 17, there is no support for “contraction” or for “a cycle.” (Here is a case of something that is generally believed to be true in the world outside the GMAT but that is not actually supported by the passage.) The wording of (D) is convoluted and dense, but the passage supports everything here: “statistical evidence at hand” could well refer to “0.01 percent,” “six-sevenths,” or “three-quarters.” The author is indeed “judging the veracity of a scientific model.” The passage discusses the creation of the universe and its behavior, a “process which cannot be repeated in a laboratory.” But, more important than any of that is the fact that if the author didn’t believe (D), there’s no way he could say that the Big Bang model is valid. (D) is correct. (E) is a distortion of the last sentence. The author may agree that the Big Bang model is simple but not that this is what “accounts for its accuracy.” According to paragraphs 1 and 4, it is the model’s match with observation that demonstrates its validity. The correct answer is (D).

12.   (E)

According to the passage, a helium nucleus

Step 2: Analyze the Question Stem

The phrase “according to” announces this as a Detail question, and the question gives us a clear reference for our research: “helium nucleus.”

Step 3: Research the Relevant Text

We know that the passage grows increasingly detail oriented in the third and fourth paragraphs. A quick scan reveals mention of the helium nucleus in line 16. The key to researching a detail is its context. Here, the sentence on the helium nucleus (lines 15–16) tells us that it is formed of two deuterons. And the sentence right before that tells us that deuterons are made up of one neutron and one proton.

Step 4: Make a Prediction

Our research shows that a good prediction for the correct answer choice is one that says the helium nucleus is made up of two deuterons or two protons and two neutrons.

Step 5: Evaluate the Answer Choices

The correct answer, (E), uses almost exactly the language of our prediction. (A) is too extreme. The passage does say that fusion stopped after the first few minutes. But it doesn’t say that fusion could never happen under any other circumstance. Moreover, (A) distorts the passage, which claims that “fusion into heavier elements” stopped—not that fusion into helium itself stopped. (B) is a tricky distortion. The sentence following the reference to the helium nucleus says, “As the universe continued to expand and cool, its density became too low …” But “its” refers to the universe, not the helium nucleus. Both (C) and (D) are mentioned nowhere in the passage. (D), interestingly, happens to be a fact—you may recall from high school chemistry that a helium nucleus has four times the mass of a hydrogen nucleus. However, that fact is not mentioned in this passage, and it is not the correct response to a question asking for what’s true “according to the passage.” This answer is a good example of our caution against bringing in outside knowledge. Choice (E) is correct.

On the left, we’ve shown how keywords help you to identify the major elements of the passage and its structure and what you could skim over. On the right, we’ve shown what you might be thinking as you read the passage strategically.

Passage for Questions 13–17

Analysis

Parfleche is the French name for the Plains peoples’ hoemskot ’eo—an envelope-shaped rawhide container for storing clothes, food, and personal items. The parfleche served not only as a practical and durable storage solution, but also as a decorative object of spiritual significance. Among certain tribes, most notably the Cheyenne and Arapaho, parfleches were decorated by the women’s painting society, whose members among the Cheyenne were known as moneneheo, the Selected Ones. Although similar in their economic and social importance to craftsmen’s guilds in medieval and Renaissance Western Europe, the painting society also had a spiritual or religious nature. The shamanistic society required application for admission and held its members to high artistic and moral standards. The society further displayed its importance by defining aspects of Cheyenne wealth and status.

The topic, the parfleche, is addressed and defined in the first sentence.

The “not only … but also” structure indicates that the author will focus on the spiritual significance of the parfleche.

Another contrast: the group that made parfleche was economic/social “but also” spiritual/religious.

Painting on rawhide was fraught with challenges. If painting was attempted while the prepared hide was too moist, the applied paint bled, but if the hide was too dry, the skin did not absorb the pigments. This restricted the time frame in which painting could best be completed, which meant that designs had to be visualized fully before the work started. Moreover, every aspect of creating a parfleche was a sacred act. Each design element, for instance, was a syntagma—a linguistic or visual unit intended to convey meaning—freighted with symbolic referents. For example, diamond shapes represented the ha ’kot, the grasshopper, an abundant grass eater itself symbolic of the bison, the sacred source of food, shelter, tools, and clothing. The tools used were also symbolic: the shape of the “flesher” used to prepare the hide represented lightning bolts—emblematic of the masculine essence of spirit. The flesher removed the flesh from the hide, transforming it into a spiritual container that would hold earthly matter (the people’s material goods).

Making of parfleche: difficult; highly symbolic; even tools symbolic.

Even the position of the parfleche in the lodge held symbolic significance. It was stored beneath the bed of older women, not only because they were careful guardians, but also because they were closer to Grandmother Earth, from whose union with the lightning spirit the animals and plants of the middle world came to provide food and shelter. The symbolism of every aspect of the parfleche, therefore, from the interpretable design work on its outside to its storage place within the lodge, reflects the Cheyenne belief in a complementary worldview: the blending of the masculine spirit and the feminine physical matter.

Parfleche kept by mature women (considered spiritual people). Conclusion: parfleche reflects masculine/feminine blend.

Passage Map

¶1: Intro to the parfleche and the people who created it

¶2: The process and symbolism involved in making the parfleche

¶3: The parfleche as a symbol of the Cheyenne worldview

Topic: The parfleche

Scope: The spiritual significance of the production, decoration, and use of the parfleche

Purpose: To explain the cultural and spiritual significance of the parfleche to the Plains peoples

13.   (E)

According to the passage, one reason why there was a limited time frame in which to paint a prepared rawhide was that

Step 2: Analyze the Question Stem

The phrase “According to the passage …” indicates that this is a Detail question. The stem gives us a clear reference for our research: “limited time frame in which to paint a prepared rawhide.”

Step 3: Research the Relevant Text

Using our Passage Map, we can refer to the second sentence of the second paragraph, where the author was discussing the difficulties in creating the parfleche.

Step 4: Make a Prediction

The second sentence of paragraph 2 provides everything we need for a prediction of the correct answer: “If painting was attempted while the prepared hide was too moist, the applied paint bled, but if the hide was too dry, the skin did not absorb the pigments.”

Step 5: Evaluate the Answer Choices

(E) agrees perfectly with the first half of this statement. (A) is unsupported by the passage. (B) and (C) are both 180 wrong answer traps, since the sentence actually said that when the hides are too dry, they would not absorb paint. (D) is a distortion; it reverses the cause-and-effect relationship described in the passage. The reason that the designs had to be visualized in advance was the limited time during which the hides could be painted, not the other way around. Choice (E) is the correct answer.

14.   (E)

The main purpose of the passage is to

Step 2: Analyze the Question Stem

The wording here—“main purpose of the passage”—makes this one of the most common Global question stems that appear on the GMAT.

Step 3: Research the Relevant Text

Since we paraphrased the author’s purpose while creating our Passage Map, we can simply refer to that; there is no need to go back to the passage itself.

Step 4: Make a Prediction

The Passage Map tells us that the author wrote this passage to explain the parfleche and its “spiritual significance” to certain Indian tribes.

Step 5: Evaluate the Answer Choices

(E) best summarizes this and properly reflects the author’s tone. Notice that (E) doesn’t use the word “parfleche” but rather its definition. Looking for the right words in Reading Comp answer choices is misguided; always look for the answer choice that matches the meaning or idea that you’ve predicted. (A) is too narrow in scope; it reflects only one detail from the text instead of the passage as a whole. (C) is too broad in scope; our author is focused on one aspect of Cheyenne culture. (B) and (D) are not correct because the author of this passage does not take a stand on or disagree with anyone about the symbolism of the parfleche or the methods used for studying it. Choice (E) is the correct answer.

15.   (D)

According to the author, the Cheyenne women’s painting society was unlike Western European guilds of the Middle Ages and the Renaissance in that

Step 2: Analyze the Question Stem

This is another Detail question, again with a clear reference point for research during the upcoming Step 3. Here, we are asked what, in the author’s words, distinguished the Cheyenne women’s society that made the parfleche from medieval guilds.

Step 3: Research the Relevant Text

As expected, the contrast is highlighted for us by a keyword—in this case “Although”—in the passage. The Cheyenne group was similar to guilds in economic and social importance, but different because the Cheyenne painters’ group was “spiritual or religious.”

Step 4: Make a Prediction

The correct answer will highlight the Cheyenne groups’ spiritual side.

Step 5: Evaluate the Answer Choices

Thus, (D) is the correct answer choice. (A) is unsupported by this passage, since while we know that the women’s painting society required application for membership, we do not know that the guilds were any different in their requirements. (B), (C), and (E) all describe ways in which the women’s painting society was similar to the Western European guilds. Choice (D) is correct.

16.   (A)

You can most reasonably conclude that the Cheyenne definition of the term moneneheo (line 6) reflects

Step 2: Analyze the Question Stem

The wording of this question stem is a bit unusual, but a moment’s reflection tells us that this is an Inference question. Inference questions ask what must be true based on the passage. In this case, the passage gives us information indicating why the Cheyenne would refer to the women’s painting society members as moneneheo. The question simply wants us to paraphrase that information accurately.

Step 3: Research the Relevant Text

The question stem refers to the word moneneheo in the line 6, where the author tells us that it refers to the members of the women’s painting society and translates as “the Selected Ones.” The author goes on to explain the society’s high importance and the “high artistic and moral standards” required for admission and membership.

Step 4: Make a Prediction

Predicting an answer to the question, we would say that the Cheyenne term reflects respect for the society member’s status, talent, importance, etc. Remember, we aren’t trying to predict the words of the answer but, rather, its meaning.

Step 5: Evaluate the Answer Choices

Choice (A), with its emphasis on status, matches our prediction nicely. Although we are told that the moneneheo is a shamanistic organization, we learn nothing about its origins, so (B) is unsupported. Choice (C) requires a leap from the use of “selected” to the notion that the gods do the selecting. Nothing the author writes supports this leap. The author mentions no sense in which the term moneneheo refers to anything “self-restrictive” on the part of the women’s painting society; (D) is unsupported. The author draws no association between the term and the skinning tool or “flesher” (which is, in any case, not mentioned until the end of paragraph 2); thus, (E) is also unsupported. Choice (A) is correct.

17.   (D)

The author describes the symbolic meanings of the diamond shape most likely in order to

Step 2: Analyze the Question Stem

This is an archetypal GMAT Logic question. We are referred to a detail—here, the “diamond shape”—and asked why the author includes it in the passage. Notice that the wording of the question, ending with “in order to …” means that the answer choices must begin with verbs.

Step 3: Research the Relevant Text

The diamond shape was among the symbols mentioned in the second paragraph. It is located in a sentence that begins with the illustration keywords “For example.” The sentence goes on to tell us that the diamond represents the grasshopper, which in turn represents the bison, which in turn is sacred, and so on.

Step 4: Make a Prediction

Our research tells us that this answer must begin with a verb meaning “illustrate” or “give an example of” and continue by saying something about the symbolic meanings of the parfleche.

Step 5: Evaluate the Answer Choices

Choice (D) matches the prediction and is correct. Given the verbs that begin each answer choice, only (C) and (D) are really in the running. (C) distorts the purpose of the diamond-shape example beyond what we read in the passage. Closer reading shows answers (A), (B), and (E), already suspect for having the wrong purpose verbs, to be even worse. (A) addresses the value of the parfleche to its owner, a topic touched on nowhere in the passage. (B) suggests an unsupported, and rather ridiculous, comparison between the grasshopper and bison as characterized in Cheyenne culture. (E) tries to apply the diamond-shape example to the tool-use example that comes later in the paragraph. (D) is the correct answer.

On the left, we’ve shown how keywords help you to identify the major elements of the passage and its structure and what you could skim over. On the right, we’ve shown what you might be thinking as you read the passage strategically.

Passage for Questions 18–23

Analysis

Prior to the nineteenth century, both human and animal populations were limited by the finite resources (such as food) to which they had access. When the enormous increases in prosperity ushered in by the Industrial Revolution essentially freed many Western nations from these constraints, scientists of the time expected a Malthusian explosion in population. However, an inverse relationship between prosperity and reproduction was soon noted; the average size of families fell. The trend continues to this day and has spread to recently industrialized portions of the world.

The first sentence announces the topic: population size.

“However” signals a discrepancy between what scientists expected (a population explosion) and what really happened (a decrease in family size). We can expect the rest of the passage to present one or more explanations for this discrepancy.

Early biologists tried to explain the transition to smaller families by drawing comparisons to the animal world. Animals that have many young tend to live in hostile, unpredictable environments. Since the odds against any given offspring’s survival are high, having many offspring increases the chance that at least one or two of them will survive. In contrast, animals that have fewer children but invest more resources in childrearing tend to live in stable, less hostile environments. While the young of these “high-investment” species enjoy the benefits of a relatively safe environment, they need to compete with animals whose young are equally unlikely to perish early in life. Therefore, the biologists observed, progeny that have acquired the skills they need to compete while sheltered by a family have an advantage over their less prepared competitors. By analogy, if people living in a prosperous environment produced only a few, pampered children, those children would out-compete the progeny of parents who had stretched their resources too widely.

Here comes the first explanation, put forth by early biologists, who compare patterns of human reproduction to different animal species’ reproduction strategies.

The biologists argue that animals living in environments with few resources have many offspring, while animals living in richer environments have fewer, pampered offspring.

The biologists claim that the number of offspring humans have also depends on how resource-rich their environment happens to be.

Critics of this theory argue that there are limitations in conflating animal and human behavior. They argue instead that changes in social attitudes are adequate to explain this phenomenon. To a family in a society that is tied to the land, a large number of children is a great boon. They increase family income by being put to work early, and usually some can be persuaded to care for their parents into old age. As a society becomes richer, and as physical labor becomes less important, education may extend into the early twenties, making children economically unattractive as they now consume family assets rather than produce them. Meanwhile, plans such as pensions and Social Security mitigate the need for children to care for their parents into their dotage.

The third paragraph introduces an opposing point of view. Critics of the early biologists argue that shrinking family size in the context of newfound prosperity has more to do with changes in social attitudes than with natural selection.

The passage ends without introducing the author’s own perspective. We’re simply left with two competing explanations for the unexpected decline in family size after the Industrial Revolution, the biologists’ and the critics’.

Passage Map

¶1: Population growth paradox: scientists’ expectations vs. reality

¶2: Early biologists’ explanation: family size depends on how resource-rich the environment is

¶3: Critics’ explanation: changes in social attitudes cause family size to decline

Topic: Family size/population growth

Scope: Why industrialized, prosperous nations have shrinking families

Purpose: Describe two different explanations for why families tend to decrease in size with increasing prosperity

18.   (D)

The primary purpose of the passage is to

Step 2: Analyze the Question Stem

This Global question asks us for the passage’s “primary purpose.”

Step 3: Research the Relevant Text

A quick look at our notes on topic, scope, and purpose will suffice. There’s no need to go back to the passage itself to answer a Global question.

Step 4: Make a Prediction

Our notes say that the author’s purpose is to describe two different explanations for why families tend to decrease in size with increasing prosperity. In evaluating the choices, it will also help to recall that the author’s point of view in this passage is entirely neutral.

Step 5: Evaluate the Answer Choices

In evaluating the choices for a “primary purpose” question, it is often useful to start with a scan of the initial verb. (D) starts with the verb “present,” which is perfectly neutral and very close to our description of the author’s purpose, which we said is to “describe” two alternate explanations. This should be the choice we read first. When we do, we find that is a perfect match for our prediction. (D) is in fact the correct answer. (A) is incorrect because the author doesn’t “criticize” anything. (B) is incorrect because events that occurred “before the Industrial Revolution” are not mentioned in the passage. (C) is incorrect because the author fails to “demonstrate” that either or both of the explanations described have any merit. (E) is incorrect because the author doesn’t “argue” for or against anything. Choice (D) is correct.

19.   (C)

According to the passage, which of the following is true of a Malthusian explosion in population?

Step 2: Analyze the Question Stem

“According to” signals that this is a Detail question. The correct answer will simply paraphrase a detail mentioned in the passage.

Step 3: Research the Relevant Text

The “Malthusian explosion in population” is mentioned in the first paragraph, where we really only learn two things about it: (1) It was “expected” by scientists at the time of the Industrial Revolution, and (2) it didn’t actually happen.

Step 4: Make a Prediction

For Detail questions, what we’ve found through our research amounts to our prediction. We need an answer choice that says either that a Malthusian population explosion was expected, or that it did not actually occur, or both.

Step 5: Evaluate the Answer Choices

(C) matches our prediction and is the correct answer. (A) is incorrect because “preindustrial areas of the globe” are not mentioned in the passage. All we know is that there has not been a population explosion in industrialized nations; what has happened elsewhere is not discussed in the passage. (B) is the opposite of what we need. The first paragraph of the passage states that a Malthusian population explosion, though believed to be inevitable during or after the Industrial Revolution, did not actually occur. As for (D), the closest thing to “social scientists” mentioned in the passage would be the “critics” of the early biologists. However, the passage does not state whether there is a “consensus” among these critics’ views or when such a “consensus” was arrived at. Finally, (E) is a distortion. The population explosion was avoided, according to the passage, for one of the two possible reasons put forth by the “early biologists” and their “critics,” respectively. While the critics’ explanation does include extended education as an economic liability that helps convince people in industrialized nations to have fewer children, the author of the passage does not endorse the critics’ explanation as correct. Choice (C) is the correct answer.

20.   (A)

The last paragraph performs which of the following functions in the passage?

Step 2: Analyze the Question Stem

This question asks us for the function of the final paragraph and is thus a Logic question. Logic questions are essentially code for “Why did the author bother to include this part of the passage?”

Step 3: Research the Relevant Text

Because we are being asked for the function of the third paragraph in the passage as a whole, we should cast a quick glance at our Passage Map. The first paragraph introduces the curious fact that family size has declined in industrialized nations, the second paragraph gives the early biologists’ explanation for this phenomenon, and the third paragraph presents the critics’ explanation.

Step 4: Make a Prediction

The author’s point of view in this passage is completely neutral; there is no endorsement of one or the other explanation for decreasing family size. The third paragraph, then, simply describes one of two possible explanations.

Step 5: Evaluate the Answer Choices

(A) matches our prediction and is the correct answer. The “phenomenon described in the first paragraph” is the decline in family size in industrialized nations. (B) is incorrect because the function of the third paragraph is simply to present another explanation. The author of the passage doesn’t “criticize” either explanation (though the critics do). (C) is incorrect because while the critics’ argument is precisely that social attitudes change as societies become wealthier, this is again not necessarily the author’s opinion. We must always keep the author’s perspective in mind when answering Logic questions. (D) is incorrect because the “phenomenon” explained in the third paragraph was originally presented in the first paragraph, not the second. Finally, (E) is incorrect for the same reason that (B) and (C) are incorrect: the idea that changing social attitudes are sufficient to explain decreasing family size in industrialized nations is not endorsed by the author. The third paragraph simply presents the critics’ argument; the author doesn’t evaluate that argument at all. Choice (A) is the correct answer.

21.   (E)

The passage mentions each of the following as a possible reason average family size might fall in recently industrialized nations EXCEPT:

Step 2: Analyze the Question Stem

This question asks for the one answer choice not specifically mentioned in the passage, so we have a Detail EXCEPT question.

Step 3: Research the Relevant Text

We’ll need to look up each answer choice to rule out those that are mentioned in the passage. The question stem directs us to the second and third paragraphs, as those paragraphs contain the explanations for why family size tends to fall in industrialized nations.

Step 4: Make a Prediction

For Detail EXCEPT questions, we’ll have to look up the answer choices individually. We’re looking for that answer choice that is not mentioned in the passage.

Step 5: Evaluate the Answer Choices

(A) is incorrect because lengthy education that makes children a drain on family resources is mentioned in the third paragraph as part of the critics’ explanation. (B) is incorrect because advantaged children out-competing disadvantaged ones comes up at the end of the second paragraph as part of the early biologists’ explanation. (C) is incorrect because improved social care of the elderly is mentioned during the critics’ explanation in the third paragraph. The “changed social attitudes” in (D) form the crux of the critics’ explanation, so (D) is incorrect. (E) must, by default, be the correct answer. Indeed, the second-to-last sentence in the third paragraph states that in industrialized societies, physical labor becomes less important, not more so. (E) states the opposite of what the passage says and is therefore correct.

22.   (E)

The information in the passage suggests that which of the following animals would be most likely to have many young?

Step 2: Analyze the Question Stem

The word “suggests” in the question stem signals an Inference question.

Step 3: Research the Relevant Text

The question stem directs us to the early biologists’ comparison, in the second paragraph, of human family sizes to the reproductive patterns of animals living in different environments. We learn that animals that have many young “tend to live in hostile, unpredictable environments,” while animals that have fewer young “tend to live in stable, less hostile environments” and “invest more resources in childrearing.”

Step 4: Make a Prediction

The question asks for an animal that has many young. We can expect the correct answer to describe an animal that lives in a harsh environment. Moreover, while the passage does not explicitly say so, we can infer that the animal with many young will provide minimal care for its offspring.

Step 5: Evaluate the Answer Choices

(E) matches our prediction and is the correct answer. An animal that lives in “seasonal streams and lakes” certainly lives in an unpredictable environment, since those bodies of water are prone to drying up. Moreover, the presence of many predators certainly contributes to a very harsh environment. (A) might look tempting because a drought-susceptible grassland definitely qualifies as a hostile and unpredictable environment, but we would not expect an animal that has many offspring to be “fiercely protective” of those offspring. (A) is thus a perfect example of a half-right/half-wrong answer choice. (B) is incorrect because while the islands are threatened by human encroachment, they may nevertheless constitute a stable, friendly natural environment. (C) is incorrect because even though the meat eater has to migrate in search of food, there is no suggestion that food is ever unavailable, and the lack of predators implies a non-hostile environment. (D) is incorrect because little competition again suggests a non-hostile environment. Choice (E) is correct.

23.   (A)

The author mentions a decrease in the importance of physical labor (line 28) in order to

Step 2: Analyze the Question Stem

The phrase “in order to” indicates that this is a Logic question. Specifically, we are asked why the author mentions a decrease in the importance of physical labor.

Step 3: Research the Relevant Text

The question stem sends us to line 28, which is in the middle of the third paragraph. That paragraph begins with the critics’ argument, which is that “changes in social attitudes are adequate to explain” shrinking family size in industrialized nations. The rest of the paragraph contains the critics’ evidence for their position.

Step 4: Make a Prediction

Physical labor’s diminishing importance is a social phenomenon that helps to bring about a change in social attitudes, so it is essentially a piece of evidence for the critics’ argument.

Step 5: Evaluate the Answer Choices

(A) matches our prediction and is the correct answer. (B) is a distortion. The critics’ argument counters that of the early biologists, not the argument of those scientists alive at the time of the Industrial Revolution who predicted a Malthusian population explosion. (C) is another distortion. It’s family size that the critics contest must change to meet the demands of a changing economy, not family “structures.” (D) is incorrect because the author doesn’t rebut any argument; the tone of the passage is perfectly neutral. Finally, (E) is another distortion. The critics argue that large families can increase family income in preindustrial societies “tied to the land.” But a decrease in the importance of physical labor happens in industrial societies, not in preindustrial ones. Choice (A) is correct.

On the left, we’ve shown how keywords help you to identify the major elements of the passage and its structure and what you could skim over. On the right, we’ve shown what you might be thinking as you read the passage strategically.

Passage for Questions 24–29

Analysis

Generally, interspecific matings represent an evolutionary dead end, producing sterile offspring, if any at all. For some species of birds, however, such pairings may indeed bring evolutionary advantages to the participants. In the case of the female collared flycatchers of Gotland, three distinct factors may work to make interspecific pairings with pied flycatcher males reproductively beneficial.

“Generally” implies that there will be an exception. Usually, interspecific mating doesn’t work.

The female collared flycatcher is the exception. We’re going to see three reasons why.

In many instances, female collared flycatchers nest with male pied flycatchers while continuing to mate with other collared flycatchers, in effect parasitizing the pied flycatchers, who invest in rearing and fledging any offspring. Often, more than half of the offspring raised by interspecific flycatcher pairs are, in fact, not hybrids. Furthermore, an estimated 65 percent of the hybrid offspring of the resident pied flycatcher male are male. Because hybrid females are sterile and males are not, this male bias minimizes the primary disadvantage of interspecific matings: sterile offspring. Habitat specialization may be a third mechanism: these pairings tend to occur in the late spring when the coniferous woods favored by the pied flycatcher provide a greater availability of food than the deciduous woods where the collared flycatchers tend to live. Together, these factors form a mechanism to improve substantially the reproductive success of female collared flycatchers beyond what would be expected of interspecific mating with pied flycatcher males.

Why interspecific mating works for female collared flycatchers:

  1. They get pied flycatchers to raise their collared flycatcher offspring.

  2. The offspring aren’t all sterile.

  3. Pied flycatchers live where the food is.

These factors seem to work together.

Although all three of these mechanisms appear to act in concert to form a single elaborate mechanism specifically evolved to circumvent the usual disadvantages of interspecific mating, studies have shown similar motivations for the behavior of female collared flycatchers mating within the species. According to Professor Siever Rohwer, collared flycatcher females will choose to nest with subordinate collared flycatcher males that inhabit good territory because collared flycatcher females must pair-bond in order to be successful in raising offspring. To engender the best offspring, however, the females will continue to copulate with higher-quality collared males with whom they are not paired. Thus, females seem to be nesting with males of any species with the best territories available at the time, but they will continue to mate with more attractive males outside of their pair bonds.

“Although … similar motivations”: female collared flycatchers act the same way when mating with male collared flycatchers.

Pair with males that have good nests; mate with others, too.

A highly unusual behavior, interspecific mating seems to provide certain reproductive advantages to the collared flycatcher female. However, it remains unclear whether the mating behavior of female collared flycatchers evolved to circumvent the usual problems with interspecific mating or whether the behavior is simply an extension of how female collared flycatchers behave when mating within their own species.

The author is wrapping up.

“However” shows the central unanswered question: did female collared flycatchers evolve this way because of interspecific mating or just because they act this way generally?

Passage Map

¶1: Interspecific mating—usually evolutionary dead end/sterile offspring; FCF is exception

¶2: 3 advantages of interspecific mating for FCF

¶3: FCF nests/mates same way intraspecifically, too. Same advantages

¶4: Question—did FCF evolve this way for interspecific mating or just in general?

Topic: Interspecific mating—female collared flycatcher exception

Scope: FCF’s interspecific and intraspecific mating behaviors and their evolutionary advantages

Purpose: Describe FCF’s mating/nesting behavior and pose an unanswered question about its origins

24.   (D)

The author’s primary purpose is to

Step 2: Analyze the Question Stem

“Primary purpose” signals that this is a Global question.

Step 3: Research the Relevant Text

No research within the passage is required. We just need to consult our Passage Map, specifically our statement of the author’s purpose.

Step 4: Make a Prediction

Our statement of the author’s purpose said that the author was describing the mating behavior of the female collared flycatcher and posing a question about it. Two possible explanations are presented: (1) Interspecies mating brings distinct benefits, and (2) interspecies mating is just an extension of normal behavior. With that background, we’re more than ready to assess the choices.

Step 5: Evaluate the Answer Choices

(D) best summarizes the author’s purpose. (A) is incorrect because the author never takes a stand on either explanation and so does not “criticize” anything. Similarly, (B) and (E) are wrong because the author does not “defend” anything. The author never calls for further research, so (C) is not correct either.

25.   (C)

According to the passage, female collared flycatchers’ mating with male pied flycatchers could be explained by any of the following reasons EXCEPT:

Step 2: Analyze the Question Stem

“According to the passage” is a clear indication that this is a Detail question. Just make sure you didn’t miss the “EXCEPT” at the end of the stem. Here, the four wrong answers will all have been cited in the text. The correct answer will not. Remember, the correct answer to a Detail EXCEPT question may contradict the text or not be mentioned in the passage. The right answer might also repeat a piece of information that appears in the text but that has no relevance to the question posed in the stem (i.e., make a faulty use of detail). Here, the wrong answers will give possible explanations for the female collared flycatcher’s unusual mating behavior.

Step 3: Research the Relevant Text

The female collared flycatcher’s nesting and mating behaviors are explained in the second and third paragraphs. While the question asks specifically about explanations for the interspecific mating behaviors, remember that the third paragraph told us that the motivations for intraspecific mating behaviors were similar.

Step 4: Make a Prediction

We cannot predict what the correct answer will say, but we know that the four wrong answers will be supported by text from paragraphs 2 or 3.

Step 5: Evaluate the Answer Choices

The coniferous forests where the pied flycatchers live are indeed richer in food late in the season, as (A) suggests. Females may use the pied males for their help in rearing young—including those sired by other males—as (B) says. But as for (C), the passage says that pied males mating with collared females father more males than females. So, (C) directly contradicts paragraph 2, making (C) the correct answer. (D) and (E) are both supported by paragraph 3. Choice (C) is correct.

26.   (C)

The bias toward male offspring resulting from the mating of collared flycatcher females and pied flycatcher males is presented as evidence that

Step 2: Analyze the Question Stem

The language of the question stem indicates that this is a Logic question. This stem goes out of its way to be helpful, though, by telling us that the detail at issue—the predominance of male offspring from male pied and female collared flycatcher mates—is presented as evidence. Our job will simply be to research what it is used as evidence for.

Step 3: Research the Relevant Text

The reference to the offspring of male pied and female collared flycatcher mates leads us to paragraph 2. We’re told that the male offspring are not sterile, thus minimizing a typical downside to interspecies mating.

Step 4: Make a Prediction

The correct answer will tell us that the male offspring bias is evidence that a typical disadvantage—sterile offspring—of interspecies mating is not present when female collared flycatchers and male pied flycatchers mate.

Step 5: Evaluate the Answer Choices

(C) matches the prediction nicely. It is correct. (A) is simply not mentioned in the passage; the dominance of offspring is not discussed. (B) is a distortion; it is the interspecific hybrids, not pure collared flycatchers, that show a male bias in their offspring. (D) is incorrect because no argument is ever made about future generations of interspecific breeding. And (E) is incorrect because the passage does not comment on the regularity of interspecific breeding by any bird other than the female collared flycatcher. The answer is (C).

27.   (A)

It can be inferred from the passage that

Step 2: Analyze the Question Stem

Here’s an open-ended Inference question. The stem references no specific detail or part of the passage. We will begin our evaluation of the answers with reference to what we know about the topic, scope, and purpose overall.

Step 3: Research the Relevant Text

Given that there is no reference to guide our research within the passage, we should simply review our topic, scope, and purpose summaries. We may have to do further research choice by choice.

Step 4: Make a Prediction

Lacking a point of reference in the passage, we cannot make a prediction beyond saying that the correct answer will follow from the passage and agree with the author. That should be enough to help us find the correct answer or, at least, eliminate one or more of the wrong ones.

Step 5: Evaluate the Answer Choices

The second paragraph points out that the greater abundance of food in pied flycatcher habitats late in the mating season is a mechanism that makes interspecies pairings reproductively beneficial. If food were not relevant to the success of raising offspring, it could not be a mechanism to make interspecific mating reproductively beneficial. So (A) is true based on the passage; this is the exact criterion for the correct answer in an Inference question.

All four wrong answers are either unsupported by anything addressed by the passage or out-and-out contradict the passage. (B) is not mentioned—the percentage of male offspring is discussed only in relation to interspecific pairs. Mating for life is never mentioned either, so (C) is incorrect. The fact that females do choose males on the basis of reproductive benefits makes (D) incorrect; it contradicts what the passage implies. Lastly, we have no basis for knowing the proportion of females that engage in extra-pair matings, so (E) is unsupported. Choice (A) is correct.

28.   (C)

Professor Rohwer would most likely agree with which of the following statements?

Step 2: Analyze the Question Stem

This is also an Inference question, as signaled by the phrase “most likely agree with …” The key is to spot that it is Professor Rowher’s opinion that we are drawing our inference from.

Step 3: Research the Relevant Text

Rohwer’s opinion is discussed in the third paragraph, where the passage demonstrates the similarities between the female collared flycatcher’s interspecies and intraspecies mating behavior. Rowher’s point is that pair-bonding is essential to the female collared flycatcher’s success in rearing young. The implication is that the female collared flycatcher will nest with one male and mate with others regardless of whether she is engaged in intra- or interspecies mating.

Step 4: Make a Prediction

The correct answer will have to address the context in which the author cites Rowher. The point is that it may be the necessity of pair-bonding, rather than an adaptation that favors interspecific mating, that drives the birds’ behavior.

Step 5: Evaluate the Answer Choices

Our prediction supports (C) as the correct answer. (B) and (E) are unsupported: the relative benefits of studying flycatchers rather than other birds is not discussed, nor is Rohwer’s opinion on the benefits of hybridization. (A) and (D) contradict, at least by implication, the message of the passage; they also include the phrases “all traits” and “played no role,” which flag extreme statements. Choice (C) is correct.

29.   (B)

The mating behavior of female collared flycatchers paired with subordinate male flycatchers is offered as

Step 2: Analyze the Question Stem

Here, we have another Logic question. Like all Logic stems, this one gives us a clear reference to guide our research. The female collared flycatchers that pair with subordinates are mentioned in the third paragraph. Remember that Logic questions ask why the author included the detail, not what she said about it.

Step 3: Research the Relevant Text

The third paragraph is about the female collared flycatcher’s intraspecies nesting and mating behaviors. The specific fact highlighted by the author was that the female collared flycatcher behaves similarly when mating with male collared flycatchers as it does when mating with pied males. The author’s point is that the female collared flycatcher’s behavior may not have evolved exclusively in support of interspecies mating.

Step 4: Make a Prediction

While we don’t know the wording that the correct answer will use, we can anticipate that it will address the position that the female collared flycatcher’s behavior is possibly explained by more than just adaptation to interspecific mating.

Step 5: Evaluate the Answer Choices

The prediction leads us to (B). Paragraph 2 had explained how the female collared flycatcher’s behavior is well adapted to the special problems of interspecies mating, so paragraph 3 qualifies as an “alternative explanation.” Stated evidence can never be an assumption (which is unstated), so (A) is incorrect. The evidence in question does not support the adaptive explanation for interspecies breeding—in fact, it does just the opposite—so (C) is a wrong answer choice. This evidence is also not an explanation for why hybridization is a dead end; thus, (D) is wrong. Lastly, that the collared and pied flycatchers are separate species is a given in the passage; otherwise, the pairing of pied males and collared females could not be called interspecific. Moreover, this evidence has nothing to do with this issue, making (E) incorrect. The correct answer is (B).

On the left, we’ve shown how keywords help you to identify the major elements of the passage and its structure and what you could skim over. On the right, we’ve shown what you might be thinking as you read the passage strategically.

Passage for Questions 30–35

Analysis

A regimen of intrauterine AZT (zidovudine) as a means of reducing the chances of HIV transmission from mother to child was first described in a study known as Protocol 076, the results of which were published in the New England Journal of Medicine in November 1994. The trial found that administration of AZT to HIV-positive women during pregnancy and delivery, and to their babies after birth, reduced the transmission of HIV to the infants by two-thirds, compared with a placebo. The study was acclaimed as one of the first successful instances of a prophylaxis preventing HIV transmission—particularly “vertical” transmission, or transmission from mothers to infants. The study, prematurely ended so that all the subjects on placebos could be switched to AZT, led to the recommendation that all pregnant women with HIV take AZT.

An early HIV-prevention study.

AZT reduced transmission from HIV-positive mother to her child.

Study led to recommendation for all HIV-positive pregnant women to take AZT.

The validity of the study’s results, however, is debatable. Since Protocol 076 examined only women who had fairly high numbers of T-cells—the white blood cells that coordinate immune response—and who had previously taken AZT for less than six months, the same regimen might not succeed for other pregnant women with HIV. Moreover, the study administered large doses of AZT without examining whether lower doses, more economically feasible for uninsured or underinsured women, might be effective. Further, the trial design did not account for important variables: in particular, whether the subjects’ viral load—the amount of HIV in their bodies—might have contributed to the difference in transmission rates. Since the exact mechanism and timing of vertical transmission of the HIV virus are unknown, it cannot be ruled out that viral load influences rates of vertical transmission.

Author’s not so sure about this study.

Several reasons to doubt study: “Since,” “Moreover,” “Further.”

Author emphasizes viral load—“in particular”—as a factor that could reduce effectiveness of AZT.

Also, the study did not consider the long-term impact of AZT, which is toxic, on the babies themselves. There have been few studies of AZT’s effect on HIV-positive infants and none on its effects on uninfected babies. Since the majority of the babies born to HIV-positive women are HIV-negative, and since HIV may mutate into drug-resistant strains following a break in medication, the value of a short-term “zap” with AZT becomes suspect.

“Also”—this means we’ll see more criticism of the study.

The study failed to look at the effects of AZT on the babies.

Author concludes that value of AZT is “suspect.”

Passage Map

¶1: AZT—early study said HIV-positive pregnant women should take it; prevent transmission to child

¶2: Study flawed—only used on those with high T-cell counts, ignored(?) lower dosage, and, especially, ignored viral load

¶3: Study flawed II—ignored effect on babies; Conclusion—AZT maybe not so great

Topic: AZT and HIV transmission prevention

Scope: A study that recommended AZT use

Purpose: Critique the study in order to question AZT’s value

30.   (D)

The author questions the results of Protocol 076 for which of the following reasons?

Step 2: Analyze the Question Stem

This is a Detail question, but with slightly unusual wording. We don’t see “According to the passage,” but we can see from the way that the question is asked that we will find the correct answer stated somewhere in the text. The research clue is “The author questions the results …”

Step 3: Research the Relevant Text

We know that the author gave reasons to doubt the validity of the study in paragraphs 2 and 3. If we’ve made a Passage Map, we should have these reasons summarized on our noteboard.

Step 4: Make a Prediction

It’s hard to say exactly which of the reasons for questioning the study the correct answer will cite, but we can be certain it will be one of the following: the study used only high T-cell patients, it ignored lower dosage possibilities, it ignored the patients’ viral loads, or it failed to study the effect of AZT on the babies.

Step 5: Evaluate the Answer Choices

The passage states that the potential effects of viral load, or the amount of HIV in patients’ bodies, were not considered in this study (lines 18–20). So (D) is correct. Don’t be misled by the fact that the answer didn’t use the term “viral load.” It instead uses the term’s definition, which the passage provides in the same sentence that mentions “different transmission rates.”

(A) is a distortion. The passage implies that the women in the study were, in fact, selected on the basis of T-cell counts. The author was unhappy that it was limited to those with high counts. (B) is a faulty use of detail. It is true that the trial was discontinued and all subjects were shifted to AZT, but the author doesn’t present this fact as a problem with the study. (C) is simply not mentioned. We’re told that the majority of babies born to HIV-positive women are born HIV-negative, but the 75 percent figure comes out of nowhere. (E) is another faulty use of detail. The author does state that the mechanism and timing of vertical transmission (from mother to child) are unknown, but he does so in order to emphasize why viral load is such an important issue, not as grounds for criticizing the study. The answer is (D).

31.   (E)

The author of this passage would be LEAST likely to challenge the benefits of intrauterine AZT for which of the following HIV-positive women and/or their future children?

Step 2: Analyze the Question Stem

This is an Inference question with an important twist. We are used to seeing questions that ask what the author would “most likely agree with,” but this question asks us to apply what we’ve learned about the author’s opinion. We’ll research with an eye to language that tells us the type of patient for whom AZT is most likely to work.

Step 3: Research the Relevant Text

The passage states (in lines 12–15) that the only women included in Protocol 076 had fairly high T-cell counts and had taken AZT for less than six months. So, the author would be unlikely to dispute that a woman with a similar T-cell level and a similar history of AZT use would experience a positive outcome similar to those of the women in the study.

Step 4: Make a Prediction

We’ll look for an answer describing a woman who fits the criteria from our research step.

Step 5: Evaluate the Answer Choices

(E) matches the criteria from the second paragraph nicely. It’s the correct answer. (A) is wrong because the author says that AZT might become less effective if taken after a hiatus in use (paragraph 3). (B) is wrong because the study only looked at women with high T-cell counts (paragraph 2); AZT might not help women with low counts. (C) is unsupported: the author never discusses ethnic and socioeconomic factors. As for (D), whether or not a mother has lost a previous child to HIV, or wants a healthy child this time, is not relevant to the potential success of the treatment. The correct answer is (E).

32.   (D)

The passage implies that the “viral loads” of the subjects in the study are significant because

Step 2: Analyze the Question Stem

This Inference question (“the passage implies”) gives us a clear reference for our research. Not surprisingly, it takes us back to the author’s consideration of viral load, something he emphasized as a “particular” problem with the study’s design.

Step 3: Research the Relevant Text

The author states in paragraph 2 that “the trial design did not account for important variables: in particular, whether subjects’ viral loads—the amounts of HIV in their bodies—might have contributed to the difference in transmission rates.” In other words, the author is saying that it is possible that the women in the AZT group may have had different viral loads than the women in the placebo group.

Step 4: Make a Prediction

The correct answer will follow directly from the passage. It must address the fact that the study’s failure to account for viral load may have created an unrepresentative sample in the test group.

Step 5: Evaluate the Answer Choices

(D) expresses this idea and is the correct answer. (A) and (B) are both unsupported—the passage does not supply any information about what the relative viral loads of the women or the babies actually were. Be careful with an answer like (A); we were told that all of the subjects had high T-cell counts and the results might not be broadly applicable. Don’t confuse details; do research. (C) may be true, but it doesn’t answer the question at hand: Why was the viral load of the test subjects significant? (E) is a half-right/half-wrong answer. Viral load may indeed be a critical factor, but the researchers did not consider it in selecting test and control group subjects. Choice (D) is correct.

33.   (C)

The primary purpose of the passage is to

Step 2: Analyze the Question Stem

This is a typical Global question. Even though some questions may contain tougher answer choices than others, the question types themselves function the same way throughout the test.

Step 3: Research the Relevant Text

No textual research is required. We’ll just need to consult our Passage Map, particularly our summaries of the author’s scope and purpose.

Step 4: Make a Prediction

The most important word in our summary of the purpose is the verb. Here, we used “critique.” We may not find that exact word in our correct answer, but the verb used will have to indicate that the author is trying to caution us against accepting the completely rosy view of AZT given by the study.

Step 5: Evaluate the Answer Choices

(C) is correct. The verb phrase “raise doubts” matches our interpretation of the author’s purpose. Notice that no other answer begins with as appropriate a verb. (A) is too broad: the passage deals with only one HIV/AIDS treatment method, not all such treatments in general. (B) is wrong because, while the passage does cite problems with the design of a clinical trial, (B) is too extreme in characterizing the design as “careless” or as having “medical errors.” (D) is incorrect because while the process of vertical transmission is mentioned, it is neither described in detail nor is it the purpose of the passage. (E) is incorrect because the author is only looking at one treatment possibility— intrauterine AZT. The correct answer is (C).

34.   (B)

The author of the passage would be most likely to agree with which of the following statements?

Step 2: Analyze the Question Stem

Here’s a broadly worded Inference question. It doesn’t give us any specific clues for research, but importantly, it tells us that the correct answer is something that must follow from the author’s opinion.

Step 3: Research the Relevant Text

Lacking any exact research reference, we should go back to the Passage Map and refresh our memory of the author’s scope and purpose.

Step 4: Make a Prediction

We’ll use the author’s scope and purpose to pick out the correct answer or, at least, to eliminate choices that the author would either disagree with or have no opinion about.

Step 5: Evaluate the Answer Choices

(B) is correct: it follows from the author’s criticisms of Protocol 076, which studied only women with high T-cell counts who had been taking AZT for less than six months. The author believes this study was insufficient to recommend giving AZT to all pregnant women with HIV. (A) is wrong because the author never suggests that AZT, which is known to be toxic (paragraph 3), is probably harmless to some people. (C) is incorrect. There is no evidence that the author thinks the study should have been continued. Any complaints he makes about it involve its setup, not its termination. (D) is unsupported: the ethics of this trial are not in question. (E) is extreme. The author suggests a cautious attitude toward treating HIV-positive pregnant women with AZT, but he never rules it out entirely. Choice (B) is correct.

35.   (E)

The primary function of the second paragraph is to

Step 2: Analyze the Question Stem

This Logic question asks us for the author’s purpose in writing paragraph 2.

Step 3: Research the Relevant Text

For Logic questions that ask about complete paragraphs, the best approach is to consult your notes for that paragraph in your Passage Map.

Step 4: Make a Prediction

For paragraph 2, our Passage Map read: “Study flawed—only used on those with high T-cell counts, ignored(?) lower dosage, and, especially, ignored viral load.” The correct answer will say something along the lines of “describe flaws” or “outline shortcomings.”

Step 5: Evaluate the Answer Choices

(E), “identify problems,” accurately reflects the content of paragraph 2 and is the correct answer. All of the wrong answers miss the point of the paragraph.

GMAT BY THE NUMBERS: READING COMPREHENSION

Now that you’ve learned how to approach Reading Comprehension questions on the GMAT, let’s add one more dimension to your understanding of how they work.

Take a few minutes to read the following passage and try the questions associated with it. The next pages feature performance data from thousands of people who have studied with Kaplan over the decades. Through analyzing this data, we will show you how to approach questions like these most effectively and how to avoid similarly tempting wrong answer choice types on Test Day.

Introduced in 1978, video laserdiscs were technologically more advanced than video cassettes—they offered better picture quality without degradation over time—yet video cassettes and recorders were far more successful commercially, at least in part because relatively few movies were ever released on laserdisc. As this example illustrates, superior technology is no guarantee of success in the home audio and video market.

In home audio, vinyl records were the dominant format until the 1970s, when audio cassette tapes were introduced. Cassette tapes offered no better sound quality than vinyl records (in fact, some believed they offered lower quality), yet this format became widely successful for reasons having little to do with technical advancements in sound quality. Cassettes were more portable than records, and the ability to record from records onto cassettes made the two formats complementary. In addition to buying prerecorded records and tapes, consumers could now make copies of vinyl records and listen to them outside the home. Thus, cassette tape sales grew even as vinyl remained a popular format.

The rise of audio compact discs (CDs) was quite different. Introduced in 1983, CDs clearly offered higher sound quality than records or cassettes, yet they were not an immediate success. However, CDs were persistently and aggressively marketed by the industry, and by the 1990s they had become the most popular audio format. The dominant position of CDs was further cemented later in the 1990s by the advent of new technology that allowed consumers to create their own CDs at home—thus combining one of the best features of audio cassettes with the higher sound quality of CDs.

In home video, after the failure of laserdiscs, video cassettes remained the dominant format until the advent of digital video discs (DVDs). Introduced in 1997, DVDs quickly gained widespread popularity. These discs were based on a technology similar to that of laserdiscs and offered several clear advantages over video cassettes, including better picture quality and better search features. Yet perhaps the real key to their rapid rise was the fact that manufacturers quickly made many titles available on DVD. This combination of better technology and smart marketing helped the DVD avoid the fate of the laserdisc.

  1. The passage is primarily concerned with which of the following?

    • Contrasting the success of DVDs with the failure of laserdiscs

    • Describing the crucial role played by technology in the home audio and video market

    • Questioning the wisdom of introducing new audio or video formats

    • Illustrating that there is more than one path to success in the home audio and video market

    • Proving that good marketing is the only way to guarantee success in the home audio and video market

  2. The author of this passage would most likely agree that

    • better technology alone will never cause the success of a new audio or video format.

    • a combination of better technology and good marketing is the best way to ensure success when introducing a new format.

    • there is no perfectly reliable way to predict the success or failure of new audio and video formats.

    • companies in the home audio or video industry should invest less in product research and development than in marketing.

    • consumer behavior is inherently irrational.

  3. According to the passage, which strategy is LEAST likely to produce a successful media format?

    • Aggressively marketing a brand new format

    • Cautiously testing the market by releasing titles sparingly

    • Relying on word-of-mouth advertising

    • Creating a new format based on earlier technology

    • Introducing a product that works with existing products

Explanation: Question 1

It is crucial not to get lost in the details of a Reading Comprehension passage but to keep your eyes focused on the author’s main idea. The last sentence of the first paragraph makes it clear that the passage will focus on how superior technology does not guarantee commercial success. Each body paragraph contains an example to support this point of view. The paragraphs describe many technical details but also provide other reasons for commercial success: flexibility of use (cassettes), aggressive marketing (CDs), wide availability (DVDs). The correct answer—choice (D)—reflects the idea that success is determined by several factors.

QUESTION STATISTICS
8% of test takers choose (A)
19% of test takers choose (B)
2% of test takers choose (C)
64% of test takers choose (D)
7% of test takers choose (E)
Sample size = 2,163

Many test takers focus too much on what the details say, rather than what idea the details support. The testmakers understand this tendency, and they craft answer choices that reflect only the details. You can see how the most tempting wrong answer—choice (B)—accurately describes the details but misses the overall point that factors other than technology have an important influence. Also note how the second- and third-most popular wrong answers also miss the overall point by focusing on individual paragraphs instead of the entire passage: (A) focuses only on the fourth paragraph and (E) on the third. Although not a commonly selected wrong answer on this question, (C) is also wrong because it misconstrues the main idea, claiming that improved formats are not a good idea rather than simply insufficient for success.

Explanation: Question 2

Despite the vague wording of this question stem, the GMAT has a very specific standard for what defines the correct answer to Inference questions such as this one—and knowing that standard allows you to successfully avoid the common traps. From the testmaker’s point of view, an inference is valid only if it unambiguously must be true. So the right answer will be something that the author must agree with. If there’s any room for doubt, or if it’s possible that the author might not agree, the choice is wrong.

QUESTION STATISTICS
29% of test takers choose (A)
46% of test takers choose (B)
23% of test takers choose (C)
1% of test takers choose (D)
1% of test takers choose (E)
Sample size = 2,150

It’s easy to see why unpopular choices (D) and (E) are wrong. The author neither makes directives to companies nor casts aspersions on consumers. But why are the two most commonly selected answers wrong? The reason is that they are too extreme.

It’s very easy for an all-or-nothing statement to be possibly false. (A) claims that better technology will never cause the success of a new format. But the author only says that better technology “is no guarantee of success.” A lack of a guarantee doesn’t mean that something will never happen, so (A) is incorrect. (B) also goes too far by claiming that superior technology and good marketing is the best way to ensure success. The author clearly thinks that combining good technology with good marketing is better than having good technology alone. But this doesn’t rule out the possibility that some other approach might be better still. In fact, marketing is not mentioned as part of the reason for the success of audio cassettes.

Notice how (C) uses more cautious wording, saying that no formula for success is “perfectly reliable.” Given that the author mentions three different ways that new formats succeeded (flexibility of use, aggressive marketing, wide availability of titles), it makes sense for the author to agree that there’s no one perfect way. That’s why (C) is correct. Learn to avoid extreme language like “never” and “best” and to embrace hedged language like “no perfectly reliable,” and you’ll be well ahead of your competition and well on the way to a higher GMAT score.

Explanation: Question 3

Notice that the question explicitly tells you to base your answer on the content of the passage. Many test takers let their own opinions cloud their analysis and thus wander into traps. This question asks you to find the strategy most likely to fail. The first and fourth paragraphs both mention that the laserdisc probably failed because not enough titles were available. That makes a cautious, slow release of titles a recipe for failure, which is why (B) is correct.

QUESTION STATISTICS
3% of test takers choose (A)
70% of test takers choose (B)
18% of test takers choose (C)
6% of test takers choose (D)
3% of test takers choose (E)
Sample size = 2,122

(A) and (E) are mentioned in the passage as reasons that new formats succeeded—CDs were aggressively marketed, and cassettes complemented vinyl—so it’s no wonder that few test takers choose those answers. Neither (C) nor (D) is mentioned in the passage, which is why these choices are wrong—there’s no support for either. But you can see from the question statistics that (C) is a popular choice. That is because you might look at “word-of-mouth advertising” and think that it isn’t a sufficiently “aggressive” marketing strategy. This may be true in the real world (although if you’re a savvy buzz marketer, you might disagree), but there is nothing in the passage to suggest that a word-of-mouth marketing campaign couldn’t be aggressive. Stick with what’s supported by the passage, and you’ll be successful on Test Day.

More GMAT by the Numbers …

To see more questions with answer choice statistics, be sure to review the full-length CATs in your online resources.